Lower Limb (Gray's Anatomy Review, BRS, Lippincott)

Pataasin ang iyong marka sa homework at exams ngayon gamit ang Quizwiz!

Muscles that insert into the greater trochanter of the femur:

1) Gluteus medius 2) Gluteus minimis 3) Piriformis (upper end of greater trochanter) 4) Obturator internus

KNOW THIS:

1) If you have a posteriorly dislocated supracondylar fracture with severe compression of the popliteal artery then you can use a lateral femoral circumflex artery to ensure adequate blood supply to the leg and foot 2) The nerve located in the space between the tibia and fibula is the deep fibular (peroneal) nerve.

Muscles that insert into the lesser trochanter of the femur:

1) Iliacus + psoas (Iliopsoas)

Anterior muscles of the thigh which inserts into the patella:

1) Rectus femoris 2) Vastus medialis 3) Vastus lateralis 4) Vastus intermedius

Attachments of the ASIS

1) Sartorius muscle 2) Inguinal ligament 3) Tensor fascia lata (about 5cm away from the iliac tubercle)

Muscles that insert into the obturator internus tendon:

1) Superior gemellus 2) Inferior gemellus

KNOW THIS:

1) The fibular (peroneal) artery is a branch of the posterior tibial artery and passes in the calf between the flexor hallucis longus and tibialis posterior, making it difficult to palpate. 2) The patella can be thought of as a bone (a sesamoid bone) that develops within the tendon of the quadriceps femoris muscle. 3) In piriformis entrapment, the sciatic nerve can be compressed when the piriformis is contracted, leading to painful sensations in the lower limb. These usually involve pain in the gluteal area, posterior thigh, and leg, most frequently resembling a disc lesion at L5-S1, with compression of the S1 spinal nerve.

The great saphenous vein

Accompanies the saphenous nerve, which is vulnerable to injury when collected surgically. It is commonly used for coronary artery bypass surgery, and the vein should be reversed so its valves do not obstruct blood flow in the graft. This vein and its tributaries become dilated and varicose commonly in the posteromedial parts of the lower limb

Medial plantar nerve

*Homologous to the median nerve in the hand* Innervates the following intrinsic foot muscles: 1) First Lumbrical 2) Abductor hallucis 3) Flexor digitorum brevis 4) Flexor hallucis brevis. The mnemonic for muscles supplied by the medial plantar nerve is the "LAFF" muscles.

Other fractures that I may or may not need to know:

*Pott fracture (Dupuytren fracture)* is a fracture of the lower end of the fibula, often accompanied by a fracture of the medial malleolus or rupture of the deltoid ligament. It is caused by forced eversion of the foot. *Pillion fracture* is a T-shaped fracture of the distal femur with displacement of the condyles. It may be caused by a blow to the flexed knee of a person riding pillion on a motorcycle. *Fracture of the fibular neck* may cause an injury to the common peroneal nerve, which winds laterally around the neck of the fibula. This injury results in paralysis of all muscles in the anterior and lateral compartments of the leg (dorsiflexors and evertors of the foot), causing foot drop.

Seven tarsal bones:

-talus -calcaneus -navicular bone -cuboid bone -three cuneiform bones.

KNOW THIS:

1) The saphenous nerve does not supply motor innervation to any muscles but is a cutaneous branch of the femoral nerve. The saphenous nerve is a cutaneous branch of the femoral nerve and supplies the skin on the medial leg along the great saphenous vein. 2) The tibial nerve lies superficially in the popliteal fossa. Compression in this area would result in a loss of plantar flexion and weakness of inversion. 3) An intracapsular femoral neck fracture causes avascular necrosis of the femoral head because the fracture damages the radicular branches of the medial and lateral circumflex arteries that pass beneath the ischiofemoral ligament and pierce the femoral neck. 4) The Achilles tendon inserts upon the calcaneus bone. This tendon represents a combination of the tendons of gastrocnemius and soleus muscles. The tendon of the plantaris can insert with this tendon. 5) Muscles innervated by the superior gluteal nerve (tensor fascia lata + gluteus medius & minimus). 6) The tibialis anterior, a muscle of the anterior leg compartment, inserts upon the navicular bone and, with the tibialis posterior, is a strong invertor of the foot. 7) The medial plantar nerve innervates the abductor and flexor muscles of the big toe, the first lumbrical muscle, and flexor digitorum brevis muscle and provides sensation for the medial plantar surface and three and a half toes

KNOW THIS:

1) The sural nerve is formed in the distal posterior aspect of the leg by the convergence of the medial sural cutaneous nerve (off the tibial nerve) and the lateral sural cutaneous nerve (off the common fibular nerve). The sural nerve parallels the small saphenous vein. 2) Damage to the sural nerve would lead to numbness and paresthesia in the posterior leg, particularly to the dorsal aspect of the fifth toe and lateral malleolus of the fibula 3) The superficial fibular nerve supplies motor innervation to the lateral compartment of the leg and sensory innervation from the anterior aspect of the distal leg and dorsum of the foot. 4) The deep fibular nerve supplies motor innervation to the anterior compartment of the leg and sensory innervation from the web space between the great toe and the second toe. 5) Cremasteric reflex. The cremasteric reflex is a cutaneous reflex that tests the integrity of the L1-2 spinal cord segments. Lightly stroking the superior and medial parts of the thigh stimulates the sensory fibers of the ilioinguinal nerve (the afferent limb of this reflex), which, in turn, stimulates the contraction of the cremaster muscle to pull up the testis on the ipsilateral side. This efferent limb of the reflex is activated by the motor fibers of the genital (cremasteric) branch of the genitofemoral nerve. The genitofemoral nerve is formed by the ventral rami of L1-2 6) The calcaneal (Achilles or ankle jerk) reflex is a deep tendon reflex that confirms the integrity of the tibial nerve and the S1-2 spinal segments, from which this nerve is primarily derived 7) The tibial tuberosity is the insertion site for the quadriceps femoris 8) The saphenous nerve innervates the skin on the medial side of the leg and foot. 9) The sural nerve is derived from both the tibial and common fibular (peroneal) nerves in the proximal posterior leg. It supplies the skin on the posterior leg and lateral side of the ankle, heel, and foot. It travels in company with the small saphenous vein and is vulnerable in surgery involving that vessel. 10) The obturator nerve supplies most of the muscles in the medial (adductor) compartment of the thigh and also has a small cutaneous territory in the medial thigh. The cutaneous branches may be damaged in surgery of the upper segment of the great saphenous vein. 11) The obturator artery is a branch of the internal iliac artery within the pelvic cavity. It enters the thigh through the obturator canal and divides into anterior and posterior branches, which envelope the adductor brevis muscle. The obturator artery supplies the adductor muscles in the medial compartment of the thigh. 12) The popliteal artery is the direct continuation of the femoral artery in the popliteal fossa. It gives rise to five genicular arteries that form an extensive anastomotic network around the knee. 13) Medial femoral circumflex artery arises from the femoral or profunda femoris artery high within the femoral triangle. It supplies the muscles in the upper medial thigh and the hip joint. This important vessel is one component of the cruciate anastomosis around the hip and also provides the main supply to the head and neck of the femur. 14) The obturator artery supplies the medial compartment of the thigh. In addition, it sends a small acetabular branch into the hip joint. This vessel gives rise to the artery to the head of the femur, which follows the ligament of the head of the femur to reach the femoral head. This artery is usually pronounced in children but may or may not be present or sufficient in adults. When sufficient, it is an important collateral route for blood to reach the femoral head. 15) The adductor magnus is composed of an adductor portion (supplied by the obturator nerve that controls the adductor muscles in the medial femoral compartment) and a hamstring portion (innervated by the tibial nerve that supplies most of the hamstring muscles in the posterior femoral compartment). The biceps femoris is composed of a larger hamstring (long head) portion (supplied by the tibial nerve) and a smaller gluteal (short head) portion (supplied by the common fibular [peroneal] nerve). 16) The tibial nerve is responsible for plantar flexion of the foot at the ankle joint.

Sciatic Foramina

1. Greater Sciatic Foramen ■ Provides a pathway for the piriformis muscle, superior and inferior gluteal vessels and nerves, internal pudendal vessels and pudendal nerve, sciatic nerve, posterior femoral cutaneous nerve, and the nerves to the obturator internus and quadratus femoris muscles. 2. Lesser Sciatic Foramen ■ Provides a pathway for the tendon of the obturator internus, the nerve to the obturator internus, and the internal pudendal vessels and pudendal nerve. 3. Structures That Pass through Both the Greater and the Lesser Sciatic Foramina ■ Include the pudendal nerve, the internal pudendal vessels, and the nerve to the obturator internus

March fracture (stress fracture)

A fatigue fracture of one of the metatarsals, which may result from prolonged walking. Metatarsal fractures are also common in female ballet dancers when the dancers lose balance and put their full body weight on the metatarsals.

Pertrochanteric fracture:

A femoral fracture through the trochanters and is a form of the extracapsular hip fracture. The pull of the quadriceps femoris, adductors, and hamstring muscles may produce shortening and lateral rotation of the leg. It is common in elderly women because of an increased incidence of osteoporosis. In fracture of the middle third of the femoral shaft, the proximal fragment is pulled by the quadriceps and the hamstrings, resulting in shortening, and the distal fragment is rotated backward by the two heads of the gastrocnemius.

Bumper fracture:

A fracture of the lateral tibial condyle that is caused by an automobile bumper, and it is usually associated with a common peroneal nerve injury.

Fracture of the femoral head:

A rare injury caused by posterior hip dislocation in advanced age (osteoporosis) and requires hip replacement. It presents as a shortened lower limb with medial rotation

Ankle-jerk (Achilles) reflex:

A reflex twitch of the triceps surae (i.e., the medial and lateral heads of the gastrocnemius and the soleus muscles) induced by tapping the tendo calcaneus. It causes plantar flexion of the foot and tests its reflex center in the L5 to S1 or S1 to S2 segments of the spinal cord. Both afferent and efferent limbs of the reflex arc are carried in the tibial nerve.

Gluteal gait (gluteus medius limp)

A waddling gait characterized by the pelvis falling (or drooping) toward the unaffected side when the opposite leg is raised at each step. It results from paralysis of the gluteus medius muscle, which normally functions to stabilize the pelvis when the opposite foot is off the ground.

A 45-year-old man after being diagnosed with a posterior acetabular fracture is taken to the operating room to repair the fracture. During the neurological examination the physician notices loss of sensation to the skin of the inferior half of the buttocks, posterior, and upper medial thigh. The patient had a normal neurovascular examination preoperatively. Which of the following nerves was mostly likely damaged during the operation? A. Posterior femoral cutaneous B. Obturator nerve C. Sciatic D. Femoral E. Lateral femoral cutaneous

A. All the branches of the posterior femoral cutaneous nerve are cutaneous. It arises from the dorsal divisions of the first and second and the ventral divisions of the second and third sacral nerves and travels through the greater sciatic foramen beneath the piriformis muscle to innervate the shin over the lower parts of the gluteus maximus muscle through the inferior clunial nerves and the posterior surface of the thigh and leg and perineum via its perineal branches. [SEE IF YOU COULD FIND THIS NERVE + HELP!!]

A 55-year-old man visits the outpatient clinic complaining that he cannot walk more than 5 minutes without feeling severe pain in his feet. An image of the feet of this patient is shown in Fig. 5-10. What is the most common cause of this condition? A. Collapse of medial longitudinal arch, with eversion and abduction of the forefoot B. Exaggerated height of the medial longitudinal arch of the foot C. Collapse of long plantar ligament D. Collapse of deltoid ligament E. Collapse of plantar calcaneonavicular ligament

A. Flat foot (pes planus) is due to flattening of the medial longitudinal arch.

A 22-year-old woman is admitted with high fever and vaginal discharge. Physical and laboratory examinations reveal gonorrheal infection. A series of intramuscular antibiotic injections are ordered. Into which of the following parts of the gluteal region should the antibiotic be injected to avoid nerve injury? A. Anterior and superior to a line between the posterior superior iliac spine and the greater trochanter B. In the middle of a line between the anterior superior iliac spine and the ischial tuberosity C. Inferolateral to a line between the posterior superior iliac spine and the greater trochanter D. Inferomedial to a line between the posterior superior iliac spine and the greater trochanter E. Halfway between the iliac tuberosity and the greater trochanter

A. Gluteal injections should be given anterior and superior to a line drawn between the posterior superior iliac spine and the greater trochanter to avoid the sciatic nerve and other important nerves and vessels.

A 49-year-old man is admitted to the emergency department complaining that he has difficulties walking. Physical examination reveals that the patient suffers from peripheral vascular disease. An ultrasound examination reveals an occlusion of his femoral artery at the proximal portion of the adductor canal. Which of the following arteries will most likely provide collateral circulation to the thigh? A. Descending branch of the lateral circumflex femoral B. Descending genicular C. Medial circumflex femoral D. First perforating branch of deep femoral E. Obturator artery

A. If the femoral artery is occluded, the descending branch of the lateral circumflex femoral will provide collateral circulation to the thigh. The medial circumflex femoral artery is a proximal branch of the deep femoral artery and supplies part of the head of the femur.

A 34-year-old man is lifting heavy weights while doing squats. Unfortunately, while making a maximal effort, he drops the weight and immediately grabs at his upper thigh, writhing in pain. The man is admitted to the emergency department and during physical examination is diagnosed with a femoral hernia. What reference structure would be found immediately lateral to the herniated structures? A. Femoral vein B. Femoral artery C. Pectineus muscle D. Femoral nerve E. Adductor longus muscle

A. In a femoral hernia, abdominal contents are forced through the femoral ring, which is just lateral to the lacunar ligament (of Gimbernat) and just medial to the femoral vein. The femoral vein would be found immediately lateral to the femoral hernia. This is correct in most cases because in the majority of people, the femoral vein is found more medial to both the femoral artery and nerve in the femoral triangle.

A 15-year-old boy falls and injures his ankle while skateboarding. Examination in the emergency department leads to the conclusion that the ankle is mildly sprained, and it is wrapped with an elastic bandage. The boy still complains of pain in his ankle. Which of the following peripheral nerves is involved in carrying pain sensation from the ankle? A. Deep fibular (peroneal) B. Femoral C. Obturator D. Posterior femoral cutaneous E. Sural

A. Knowledge of Hilton's Law would lead to this correct answer. This law in a modified form, can be remembered as "a joint is innervated by the same nerves that innervate the muscles that move that joint." The deep fibular (peroneal) nerve is the only nerve listed that innervates muscles that move the ankle joint.

A 58-year-old female dancer presented to the orthopedic clinic with a complaint of pain during her work because of bilateral bunions. She was referred to a podiatric surgeon who scheduled her for surgery. The protruding bony and soft tissues of the toe were excised, and a muscle was reflected from the lateral side of the proximal phalanx, together with a sesamoid bone, upon which the muscle also inserted. What muscle was this? A. Adductor hallucis B. Abductor hallucis C. First dorsal interosseous D. First lumbrical E. Quadratus plantae

A. The adductor hallucis muscle inserts upon the lateral side of the proximal phalanx of the great toe, and also the lateral sesamoid bone, by way of its oblique and transverse heads. It is supplied by the lateral plantar nerve. The abductor hallucis inserts upon the medial side of the proximal phalanx and the medial sesamoid bone of the great toe.

A 22-year-old male martial arts competitor was examined by the clinician because of pain and serious disability suffered from a kick to the side of his knee. Physical examination revealed a dark bruise just distal to the head of the fibula. Which of the following muscles will most likely be paralyzed? A. Tibialis anterior and extensor digitorum longus B. Tibialis posterior C. Soleus and gastrocnemius D. Plantaris and popliteus E. Flexor digitorum longus and flexor hallucis longus

A. The common fibular (peroneal) nerve passes around the head of the fibula and gives off deep (L4-5) and superficial fibular (peroneal) nerve (L5, S1-2) branches. The two nerves supply the dorsiflexors and evertors of the foot, respectively. In this case, the tibialis anterior and extensor digitorum longus are the only muscles listed that are supplied by either of these nerve branches, and both are innervated by the deep fibular (peroneal) nerve.

A 42-year-old man is admitted to the emergency department after his automobile hit a tree. He is treated for a pelvic fracture and several deep lacerations. Physical examination reveals that dorsiflexion and inversion of the left foot and extension of the big toe are very weak. Sensation from the dorsum of the foot, skin of the sole, and the lateral aspect of the foot has been lost and the patellar reflex is normal. The foot is everted and plantar flexed. Which of the following structures is most likely injured? A.The lumbosacral trunk at the linea terminalis B.L5 and S1 spinal nerves torn at the intervertebral foramen C.Fibular (peroneal) division of the sciatic nerve at the neck of the fibula D.Sciatic nerve injury at the greater sciatic foramen ("doorway to the gluteal region") E.Tibial nerve in the popliteal fossa

A. The lumbosacral trunk consists of fibers from a portion of the ventral ramus of L4 and all of the ventral ramus of L5 and provides continuity between the lumbar and sacral plexuses. The deep fibular (peroneal) nerve receives supply from segments of L4, L5, and S1. It supplies the extensor hallucis longus and extensor digitorum longus, the main functions of which are extension of the toes and dorsiflexion of the ankle. L5 is responsible for cutaneous innervation of the dorsum of the foot. Injury to L4 would affect foot inversion by the tibialis anterior

The baby was quite large, and the pelvis of the mother-to-be was somewhat narrow, causing her considerable difficulty and pain during the delivery. At her specific request, it was decided to inject local anesthetic into the perineum. The genitofemoral and ilioinguinal nerves were infiltrated anteriorly, and a deep injection was made medial to the ischial tuberosity to anesthetize the pudendal nerve, which supplies much of the perineum in most cases. A few minutes later, it became very obvious to those in attendance that the injection had not been effective enough in the central and posterior parts of the perineum. A separate injection was therefore inserted lateral to the ischial tuberosity. What other nerve(s) can provide much of the sensory supply to the perineum in some individuals? A. Posterior femoral cutaneous B. Inferior cluneal nerves C. Iliohypogastric nerve D. Inferior gluteal nerve E. Middle cluneal nerves

A. The perineal cutaneous branch of the posterior femoral cutaneous nerve provides a significant portion of the cutaneous innervation of the perineum in some individuals and can require separate anesthetic blockade in childbirth or perineal surgery, if other types of anesthesia are not used.

A popliteal arterial aneurysm can be very fragile, bursting with great loss of blood and the potential loss of the leg if it is not dealt with safely and effectively. In the 18th century, Dr. John Hunter (1728-1793) discovered that if a primary artery of the thigh is temporarily compressed, blood flow in the popliteal artery can be reduced long enough to treat the aneurysm in the popliteal fossa surgically, with safety. What structure is indicated in Fig. 5-3 that is related to his surgical procedure? A. Sartorius B. Femoral vein C. Femoral artery D. Gracilis E. Adductor brevis

A. The sartorius forms the roof of the subsartorial canal (Hunter's canal), with the adductor longus and vastus medialis forming other muscular borders. Dr. Hunter mobilized the sartorius, thereby exposing the femoral artery (which continues as the popliteal artery beyond the adductor hiatus), which could be clamped while an aneurysmal popliteal artery was treated surgically. Remember that the adductor hiatus is a termination of the adductor canal and the popliteal artery arises at this site so keep the borders in mind.

A 23-year-old woman was taken to the emergency department after being involved in a head-on collision with a truck. On physical examination a hematoma was seen in the medial thigh. A CT scan revealed a fracture of the femur with a ruptured femoral artery. She was taken to the operating room for repair of the damaged structures. Two days postoperatively during physical examination the patient has loss of sensation to the anterior medial thigh and medial side of her leg and foot. Branches of which of the following nerves were most likely injured in the repair of the fracture? A. Femoral B. Saphenous C. Obturator D. Tibial E. Fibular (peroneal)

A. The skin of the anterior medial thigh and medial leg and foot is supplied by the femoral nerve. The saphenous nerve is a branch of the femoral and only supplies the medial leg and foot

A 37-year-old woman had been suffering for months from piriformis entrapment syndrome, which was not relieved by physical therapy. Part of the sciatic nerve passed through the piriformis, and a decision was made for surgical resection of the muscle. When the area of entrapment was identified and cleared, a tendon could be seen emerging through the lesser sciatic foramen, at first hidden by two smaller muscles and several nerves and vessels destined for the region of the perineum. The tendons of which of the following muscles pass through this opening? A. Obturator internus B. Obturator externus C. Quadratus femoris D. Gluteus minimus E. Gluteus medius

A. The tendon of the obturator internus leaves the pelvic cavity by passing through the lesser sciatic foramen, wrapping around the lesser sciatic notch, changing direction by about 90 degrees.

A 23-year-old man is admitted to the emergency department with a deep, bleeding stab wound of the pelvis. After the bleeding has been arrested, a magnetic resonance imaging (MRI) examination gives evidence that the right ventral primary ramus of L4 has been transected. Which of the following problems will most likely be seen during physical examination? A.Reduction or loss of sensation from the medial aspect of the leg B.Loss of the Achilles tendon reflex C.Weakness of abduction of the thigh at the hip joint D.Inability to evert the foot E.Reduction or loss of sensation from the medial aspect of the leg and loss of Achilles tendon reflex

A. The ventral ramus of L4 contains both sensory and motor nerve fibers. Injury from a stab wound could result in loss of sensation from the dermatome supplied by this segment. A dermatome is an area of skin supplied by a single spinal nerve; L4 dermatome supplies the medial aspect of the leg and foot. Loss of the Achilles tendon reflex relates primarily to an S1 deficit. The Achilles tendon reflex is elicited by tapping the calcaneus tendon, which results in plantar flexion.

The news reported that the 58-year-old ambassador received a slashing wound to the medial thigh and died from exsanguination in less than 2 minutes. What was the most likely nature of his injury? A. The femoral artery was cut at the inguinal ligament B. A vessel or vessels were injured at the apex of the femoral triangle C. The femoral vein was transected at its junction with the saphenous vein D. The medial circumflex femoral was severed at its origin E. The deep femoral artery was divided at its origin

B. The apex of the femoral triangle occurs at the junction of the adductor longus and sartorius muscles. The subsartorial (Hunter's) canal begins at this location. Immediately deep to this anatomic point lie the femoral artery, femoral vein, deep femoral artery, and deep femoral vein, often overlying one another in that sequence. This has historically been a site of injuries caused by slipping while handling a very sharp butcher's knife. For this reason, injuries at this location are referred to as the "butcher's block" injury. Fatal loss of blood can occur in just a few minutes if pressure, or a tourniquet, is not applied immediately

A 34-year-old male long-distance runner complained to the team physician of swelling and pain of his shin. Skin testing in a physical examination showed normal cutaneous sensation of the leg. Muscular strength tests showed marked weakness of dorsiflexion and impaired inversion of the foot. Which nerve serves the muscles involved in the painful swelling? A. Common fibular (peroneal) B. Deep fibular (peroneal) C. Sciatic D. Superficial fibular (peroneal) E. Tibial

B. The deep fibular (peroneal) nerve supplies the dorsiflexors of the foot, including the extensor hallucis longus and extensor digitorum longus. It also supplies the tibialis anterior, an invertor of the foot.

A 24-year-old man is admitted to the emergency department after a car collision. Radiologic examination reveals a fracture at the junction of the middle and lower thirds of the femur. An MRI examination provides evidence that the popliteal vessels were injured when the distal fragment of the fracture was pulled posteriorly. Which of the following muscles is most likely to displace the distal fracture fragment? A. Soleus B. Gastrocnemius C. Semitendinosus D. Gracilis E. Tibialis anterior

B. The gastrocnemius muscle arises from the femur just proximal to the femoral condyles. This strong muscle could displace the distal fragment of the fractured femur posteriorly. HELP!!

A 72-year-old woman is admitted to the hospital with a painful right foot. A CT scan examination reveals a thrombotic occlusion of the femoral artery in the proximal part of the adductor canal. Which artery will most likely provide blood supply to the leg through the genicular anastomosis? A. Medial circumflex femoral B. Descending branch of the lateral circumflex femoral C. First perforating branch of the deep femoral D. Inferior gluteal E. Descending genicular branch of femoral

B. The lateral circumflex femoral artery arises from the deep femoral (profunda femoris) artery of the thigh and sends a descending branch down the length of the femur to anastomose with the superior medial genicular artery and the superior lateral genicular artery. The medial circumflex femoral artery is responsible for supplying blood to the head and neck of the femur, and it does not anastomose with distal vessels at the knee.

A 34-year-old woman has a direct blow to the patella by the dashboard of the vehicle during an automobile crash. The woman is admitted to the emergency department and radiologic examination reveals patellofemoral syndrome. This type of syndrome is characterized by lateral dislocation of the patella. Which of the following muscles requires strengthening by physical rehabilitation to prevent future dislocation of the patella? A. Vastus lateralis B. Vastus medialis C. Vastus intermedius D. Rectus femoris E. Patellar ligament

B. The lower portion of the vastus medialis inserts upon the medial aspect of the patella and draws it medially, especially in the last quarter of extension—during which it is especially palpable in contraction.

A 7-year-old girl accidentally stepped on a sharp snail shell while walking to the beach. She was admitted to the hospital, where she received a tetanus shot, and the wound was cleaned thoroughly and sutured. One week later, during a return visit to her physician, it is seen that she has great difficulty in flexing her big toe, even though there is no inflammation present in the sole of the foot. Which nerve was most likely damaged by the piercing of the shell? A. Lateral plantar nerve B. Medial plantar nerve C. Sural nerve D. Superficial fibular (peroneal) nerve E. Deep fibular (peroneal) nerve

B. The medial plantar nerve innervates the abductor hallucis and both flexor hallucis longus and brevis. This nerve also provides motor supply for the flexor digitorum brevis and the first lumbrical.

A 23-year-old man is admitted to the emergency department with pain and cyanosis of his right lower limb. Doppler ultrasound studies reveal deficiency in development of his femoral artery, which appears to terminate midthigh. A thrombotic occlusion is seen in an unusual, rather tortuous, large vessel in the posterior compartment of the thigh, arising in the gluteal area and continuous inferiorly with a normal appearing popliteal artery. It is decided that a vascular graft should be placed from the femoral artery to the popliteal artery. What is the identity of the aberrant artery? A. A large, fifth perforating branch of the femoral B. An sciatic branch of the inferior gluteal artery C. Descending branch of the medial circumflex femoral D. Descending branch of the superior gluteal artery E. An enlarged descending lateral circumflex femoral artery

B. The original axial vessel of the lower limb is retained as the (usually tiny) sciatic branch of the inferior gluteal artery.

A 23-year-old man is taken to the emergency department because of anorexia, nausea, vomiting, and severe abdominal pain in the right lower quadrant. On examination, he has tenderness in the right lower quadrant with rebound tenderness. The physician suspects appendicitis. To confirm this diagnosis, the physician attempts to straighten the patient's flexed thigh. This causes the patient to wince with pain. Which of the following muscles most likely caused this symptom? A. Adductor magnus B. Psoas major C. Biceps femoris D. Obturator internus E. Gluteus maximus

B. The psoas muscle arises from the base of the transverse processes, the sides of the vertebral bodies, and the intervertebral discs, from the twelfth thoracic to the fifth lumbar vertebrae and inserted into the lesser trochanter of the femur. The psoas flexes the thigh at the hip joint on the trunk, or if the thigh is fixed, it flexes the trunk on the thigh, as in sitting up from a lying position. The inflamed appendix is pushed up against the peritoneum from the the contracted psoas. As a result it is in touch with the parietal peritoneum, producing acute pain. In some other cases it may retain the purulence of a psoas abscess, and spinal tuberculosis may present as a cold abscess in the groin. The psoas is enclosed in a fibrous sheath that is derived from the lumbar fascia. The sheath is not part of the lumbar fascia but the lateral edge blends with the anterior layer of that fascia.

A 51-year-old immigrant with tuberculosis is found to have large flocculent masses over the lateral lumbar spine. There is a similar mass located in the ipsilateral groin. Physical examination reveals increased tenderness just medial to the ipsilateral anterior superior iliac spine on palpation. This pattern of involvement most likely suggests an abscess tracking along which of the following muscles? A. Piriformis B. Psoas major C. Adductor longus D. Gluteus maximus E. Obturator internus

B. The psoas muscle arises from the base of the transverse processes, the sides of the vertebral bodies, and the intervertebral discs, from the twelfth thoracic to the fifth lumbar vertebrae and inserted into the lesser trochanter of the femur. The sheath of the psoas retains the pus of a psoas abscess, and spinal tuberculosis may present as a cold abscess in the groin (in the vicinity of the lesser trochanter). The psoas is enclosed in a fibrous sheath that is derived from the lumbar fascia. The sheath is not part of the lumbar fascia, but the lateral edge blends with the anterior layer of that fascia HELP!!!!

A 60-year-old man presents with pain on the medial aspect of his thigh. During physical examination he describes the pain to be constant, nonradiating and he also complains of numbness on the medial aspect of his leg and medial plantar arch. The nerve involved in this patient's numbness is closely associated with a structure with which of the following characteristics? A. Empties into the popliteal vein B. In its ascent in the medial aspect of the leg, it travels posterior to the medial condyle of the femur C. In its ascent in the medial aspect of the leg, it travels anterior to the medial condyle of the femur D. Arches posterior to the medial malleolus E. Is associated with nodes that drain to the horizontal group of inguinal nodes

B. The saphenous nerve is the longest and most widely distributed cutaneous branch of the femoral nerve; it is the only branch not from the sciatic nerve to extend beyond the knee. It gives sensory innervations to the medial aspect of the thigh, leg, and the medial planter arch. It accompanies the great saphenous vein over the medial side of the leg. The great saphenous vein is formed by the union of the dorsal vein of the great toe and the dorsal venous arch of the foot. It ascends anterior to the medial malleolus and passes posterior to the medial condyle of the femur and ends when it joins the femoral vein.

A 27-year-old male triathlon competitor complained that he frequently experienced deep pains in one calf that almost caused him to drop out of a regional track-and-field event. Doppler ultrasound studies indicated, and surgical exposure confirmed, the existence of an accessory portion of the medial head of the gastrocnemius that was constricting the popliteal artery. Above the medial head of the gastrocnemius, the superior medial border of the popliteal fossa could be seen. Which of the following structures forms this border? A. Tendon of biceps femoris B. Tendons of semitendinosus and semimembranosus C. Tendon of plantaris D. Adductor hiatus E. Popliteus

B. The tendons of the semitendinosus and semimembranosus provide the superior medial border of the popliteal fossa.

Tendo Calcaneus (Achilles Tendon)

Is the tendon of insertion of the triceps surae (gastrocnemius and soleus) into the tuberosity of the calcaneus.

When he attempted to lift one side of his new electric automobile from the ground to demonstrate his strength, a 51-year-old man felt a sharp pain in his back and quickly dropped the vehicle. Upon examination, it is observed that the patient has deficits in sensation on the dorsum and sole of his foot and marked weakness in abduction and lateral rotation of the lower limb. What was the nature of his injury? A. Piriformis syndrome, with entrapment of the sciatic nerve B. Disc lesion at L3-4 C. Disc lesion at L4-5 D. Disc lesion at L5-S1 E. Posterior hip dislocation

C. (LOOK AT THE DERMATOME IMAGE OF LOWER LIMB)

A 43-year-old woman is examined by a neurologist, to whom she complains of pain in her lower limb of 6 months' duration. She has pain in the gluteal area, thigh, and leg. The neurologist observes reduced sensation over the dorsum and lateral side of the involved foot and some weakness in foot dorsiflexion and eversion. A diagnosis of a piriformis entrapment syndrome is made, with compression of the fibular (peroneal) division of the sciatic nerve. Which of the following conditions did the neurologist also most likely find during her physical examination of the patient? A. Paralysis of plantar flexion B. Instability of the knee, due to paralysis of the quadriceps femoris C. Foot drop D. Spasm or clonic contractures of the adductor musculature of the thigh E. Loss of sensation in the gluteal area, by paralysis of anterior cluneal nerves

C. Entrapment compression of all or part of the sciatic nerve by the piriformis can mimic disc herniation, most commonly resembling compression of spinal nerve S1 The sciatic nerve originates from the lumbar ( (L1-L4) and sacral plexus (L4-S3) and since the piriformis is the sacral nerve (S1-S2) it makes sense that the sciatic nerve can be damaged due to piriformis entrapment. Foot drop would be anticipated with deep fibular (peroneal) nerve involvement.

A 24-year-old woman received a small-caliber bullet wound to the popliteal fossa from a drive-by assailant. The patient was admitted to the emergency department, where the surgeons recognized that the bullet had severed the tibial nerve. Such an injury would most likely result in which of the following? A. Inability to extend the leg at the knee B. Foot drop C. A dorsiflexed and everted foot D. A plantar flexed and inverted foot E. Total inability to flex the leg at the knee joint

C. Plantar flexion at the ankle would be paralyzed with the loss of the gastrocnemius and soleus, in addition to the flexors of the toes, and inversion by the tibialis posterior

A 45-year-old man is admitted to the emergency department after experiencing a sharp pain while lifting a box of books. He told the physician that he "felt the pain in my backside, the back of my thigh, my leg, and the side of my foot." During physical examination it is observed that his Achilles tendon jerk is weakened on the affected side. Which is the most likely cause of injury? A. Disc lesion at L3-4 B. Disc lesion at L4-5 C. Disc lesion at L5-S1 D. Disc lesion at S1-2 E. Gluteal crush syndrome of the sciatic nerve or piriformis syndrome

C. The Achilles tendon reflex is a function of the triceps surae muscle, composed of the gastrocnemius and soleus muscles that insert on the calcaneus. The innervation is provided primarily by spinal nerve S1. The S1 root leaves the vertebral column at the S1 foramen of the sacrum, but a herniated disc at the L5-S1 intervertebral space puts the S1 root under tension, resulting in pain and possible weakness or paralysis of S1 supplied muscles, especially the plantar flexors.

A 32-year-old man is brought to the emergency department with complaints of pain to the left ankle and knee. The patient recalls that during a football game, his left foot landed in a hole as he was running on an uneven dirt field. The ankle was externally rotated and everted while the knee twisted medially. He was unable to bear weight subsequently. During physical examination, the right ankle is swollen and there is exquisite tenderness over the right medial malleolus and the proximal lateral leg. Radiologic examination of the right lower limb reveals a displaced fracture of the neck of right fibula and a comminuted fracture of the tibial plafond and medial malleolus. Which of the following describes the most likely consequences of this injury? A. Weak "push-off" while walking and numbness over the posteromedial leg B. Weak ankle eversion and numbness over the dorsum of the foot C. High stepping gait and numbness over the dorsum and first web space of the foot D. Waddling gait and inability to feel a pin prick over the anterolateral leg E. Swing-out gait and numbness over the medial leg

C. The deep fibular (peroneal) nerve is a branch of the common fibular (peroneal) nerve and begins at about the level of the neck of the fibula, between it and the fibularis (peroneus) longus. Fracture of the head of the fibula can damage this nerve, resulting in a high stepping gait and numbness over the dorsum and first web space of the foot.

A 46-year-old woman stepped on a broken wine bottle on the sidewalk and the sharp glass entered the posterior part of her foot. The patient was admitted to the hospital, and a physical examination concluded that her lateral plantar nerve had been transected (cut through). Which of the following conditions will most likely be confirmed by further physical examination? A. Loss of sensation over the plantar surface of the third toe B. Paralysis of the abductor hallucis C. Paralysis of the interossei and adductor hallucis D. Flexor hallucis brevis paralysis E. Flexor digitorum brevis paralysis

C. The lateral plantar nerve innervates the interossei and adductor hallucis. These losses would be obvious when the patient attempts to abduct and adduct the toes.

An 82-year-old grandmother slipped on the polished floor in her front hall and was transported to the emergency department and admitted for examination with a complaint of great pain in her right lower limb. During physical examination it is observed by the resident that the right lower limb is laterally rotated and noticeably shorter than her left limb. Radiologic examination reveals an intracapsular fracture of the femoral neck. Which of the following arteries supplies the head of the femur in early childhood but no longer in a patient of this age? A. Superior gluteal B. Lateral circumflex femoral C. A branch of the obturator artery D. Inferior gluteal E. Internal pudendal

C. The obturator artery provides the artery within the ligament of the head of the femur (in about 60% of cases), the artery that supplies the head of the femur, primarily during childhood, later becoming atretic. In some individuals the medial circumflex femoral gives origin to the artery of the head. In the adult the arterial supply of the neck and head is provided by intracapsular branches of the medial circumflex femoral and lateral circumflex femoral arteries that pierce the neck of the femur.

The sartorius and gracilis muscles:

Can flex the thigh and the leg

Damage to the obturator nerve

Causes a weakness of adduction and a lateral swinging of the limb during walking because of the unopposed abductors.

Damage to the sciatic nerve

Causes impaired extension at the hip and impaired flexion at the knee, loss of dorsiflexion and plantar flexion at the ankle, inversion and eversion of the foot, and peculiar gait because of increased flexion at the hip to lift the dropped foot off the ground

Damage to the femoral nerve

Causes impaired flexion of the hip and impaired extension of the leg resulting from paralysis of the quadriceps femoris.

Anterior dislocation of the hip joint

Characterized by the tearing of the joint capsule anteriorly with movement of the femoral head out from the acetabulum; the femoral head is displaced anteroinferior to the acetabulum or the pubic bone. *The affected limb is slightly flexed, abducted, and laterally rotated*

A 48-year-old woman is admitted to the hospital with severe abdominal pain. Several imaging methods reveal that the patient suffers from intestinal ischemia. An abdominopelvic catheterization is ordered for antegrade angiography. A femoral puncture is performed. What is the landmark for femoral artery puncture? A. Halfway between anterior superior iliac spine and pubic symphysis B. 4.5 cm lateral to the pubic tubercle C. Midpoint of the inguinal skin crease D. Medial aspect of femoral head E. Lateral to the fossa ovalis

D. Femoral artery puncture is one of the most common vascular procedures. The femoral artery can be localized often by simply feeling for the strongest point of the femoral pulse just inferior to the inguinal ligament. The femoral artery can be accessed with fluoroscopic assistance at the medial edge of the upper portion of the head of the femur.

A 55-year-old woman is bitten by a dog in the dorsum of the foot and is admitted to the emergency department. The wound is cleaned thoroughly, during which it is seen that no tendons have been cut, but the dorsalis pedis artery and the accompanying nerve have been injured. Which of the following conditions would be expected during physical examination? A. Clubfoot B. Foot drop C. Inability to extend the big toe D. Numbness between the first and second toes E. Weakness in inversion of the foot

D. Injury to the dorsalis pedis artery on the dorsum of the foot can also cause trauma to the terminal portion of the deep fibular (peroneal) nerve.

A 61-year-old female immigrant had been diagnosed with spinal tuberculosis. The woman had developed a fluctuant, red, tender bulge on one flank, with a similar bulge in the groin on the same side. This presentation is likely due to spread of disease process within the fascia of a muscle with which of the following actions at the hip? A. Abduction B. Adduction C. Extension D. Flexion E. Internal rotation

D. Spinal tuberculosis can spread within the sheath of the psoas major to its insertion with the iliacus upon the lesser trochanter, presenting there also with painful symptoms. The iliopsoas muscle is the principal flexor of the hip joint. Internal rotation is performed by the adductor muscle group.

A 62-year-old man is admitted to the emergency department. Radiologic examination and the available data indicate the likelihood of a transient ischemic attack. During physical examination the ankle jerk reflex is absent. Which of the following nerves is most likely responsible for the reflex arc? A. Common fibular (peroneal) B. Superficial fibular (peroneal) C. Deep fibular (peroneal) D. Tibial E. Superficial and deep fibular (peroneal)

D. The ankle jerk reflex, elicited by tapping the tendo Achilles with the reflex hammer, is mediated by the tibial nerve.

After dividing the overlying superficial tissues and gluteal musculature in a 68-year-old female patient, the orthopedic surgeon carefully identified the underlying structures while performing a total hip arthroplasty. The key landmark in the gluteal region, relied upon in surgical explorations of this area, is provided by which of the following structures? A. Gluteus medius B. Obturator internus tendon C. Sciatic nerve D. Piriformis muscle E. Spine of the ischium

D. The piriformis muscle arises from the pelvic surface of the sacrum, passes through the greater sciatic notch, and inserts at the greater trochanter. It is considered the "anatomical key" to gluteal anatomy; the greater sciatic foramen is the "door.

A 27-year-old woman had suffered a penetrating injury in the popliteal region by an object thrown from a riding lawnmower. She was admitted to the emergency department for removal of the foreign object. After making a midline incision in the skin of the popliteal fossa, the surgical resident observed a vein of moderate size in the superficial tissues. What vein would be expected at this location? A. Popliteal vein B. Perforating tributary to the deep femoral vein C. Great saphenous vein D. Lesser (short) saphenous vein E. Superior medial genicular vein

D. The lesser (short) saphenous vein ascends up the middle of the calf from beneath the lateral malleolus, most commonly terminating at the popliteal fossa by piercing the deep fascia and joining the popliteal vein. The popliteal vein is the most superficial of major structures deep to the deep popliteal fascia.

A 42-year-old male sign painter is admitted to the emergency department after falling to the sidewalk from his ladder. Radiologic examination reveals a fracture of the proximal femur. Which of the following arteries supplies the proximal part of the femur? A. Deep circumflex iliac B. Acetabular branch of obturator C. Lateral circumflex femoral D. A branch of profunda femoris E. Medial circumflex femoral

D. The second perforating branch of the profunda femoris (deep femoral) artery commonly provides the nutrient artery to the femur, a vessel that passes through a rather large foramen to enter the proximal part of the shaft. The lateral circumflex femoral branch of the deep femoral artery supplies the vastus lateralis muscle. The medial circumflex femoral branch of the deep femoral artery supplies proximal adductor musculature and the region of the hip joint, including the neck and head of the femur. Choose second perforating branch of the profunda femoris before choice E.

In the radiographs of the knee of a male 28-year old basketball player, who had apparently suffered a tear in a medial ligament of the knee, the tubercle on the superior aspect of the medial femoral condyle could be seen more clearly than in most individuals. What muscle attaches to this tubercle? A. Semimembranosus B. Gracilis C. Popliteus D. Adductor magnus E. Vastus medialis

D. The tendinous distal portion of the adductor magnus inserts on the adductor tubercle on the upper border of the medial condyle of the femur.

The swollen and painful left foot of a 23-year-old female long distance runner is examined in the university orthopedic clinic. She states that she stepped on an unseen sharp object while running through the park several days earlier. Emergency surgery is ordered to deal with her tarsal tunnel syndrome. The tarsal tunnel is occupied normally by tendons, vessels, and nerves that pass beneath a very strong band of tissue (flexor retinaculum) on the medial side of the ankle. What is the most anterior of the structures that pass through this tunnel? A. Flexor hallucis longus tendon B. Plantaris tendon C. Tibialis anterior tendon D. Tibialis posterior tendon E. Tibial nerve

D. The tibialis posterior tendon is the most anterior of the structures that pass under the laciniate ligament (flexor retinaculum) on the medial side of the ankle to enter the sole of the foot. The structures that pass beneath the flexor retinaculum are, from anterior to posterior: Tendon of tibialis posterior; tendon of flexor Digitorum longus; posterior tibial Vessels and Nerve; tendon of flexor Hallucis longus. (This is the basis of the mnemonic: "Tom, Dick, and a Very Nervous Harry.")

The common peroneal nerve:

Divides into the deep peroneal nerve, which innervates the anterior muscles of the leg and supplies the adjacent skin of the first and second toes, and the superficial peroneal nerve, which innervates the lateral muscles of the leg and supplies the skin on the side of the lower leg and the dorsum of the ankle and foot. The sural nerve supplies the lateral aspect of the foot and the little toe

The common site for intramuscular injection of medications:

Is in *the superior lateral quadrant of the gluteal region* to avoid injury to the underlying sciatic nerve and other neurovascular structures

A 75-year-old man is admitted to the emergency department with severe pain at his right hip and thigh. An MRI examination reveals avascular necrosis of the femoral head (Fig. 5-7). Which of the following arteries is most likely injured, resulting in avascular necrosis? A. Deep circumflex iliac B. Acetabular branch of obturator C. Descending branch of lateral circumflex femoral D. First perforating branch of profunda femoris E. Ascending branch of medial circumflex femoral

E. In infants and children until about the age of 8 years, the head of the femur gets its arterial supply by a direct branch of the obturator artery (variably, the medial circumflex femoral). The arterial supply reaches the head of the femur at the fovea capitis by traveling along the ligament of the head of the femur. This source of supply is replaced later by vessels such as branches of the ascending branch of the medial circumflex femoral that pass into foramina of the neck of the femur within the capsule of the hip joint

A 60-year-old retired male marathon runner complains to his primary care physician that during his daily morning jog he experiences bouts of numbness and tingling on the medial aspect of his heel. Upon further examination the doctor discovers the patient has trouble tiptoeing and shows a positive Tinel's sign. Which of the following conditions is most characteristic of these symptoms? A. Plantar fasciitis B. Ankle inversion sprain C. Morton's neuroma D. Lateral ligament E. Tarsal tunnel syndrome

E. Tarsal tunnel syndrome is a compression neuropathy resulting from the compression of the tibial nerve in the tarsal tunnel. The tarsal tunnel is located between the medial malleolus, the inferomedial surface of the calcaneus, and the flexor retinaculum. The contents are the tibial nerve and its plantar branches, the tendons of the tibialis posterior, flexor digitorum longus, and the flexor hallucis longus muscles together with the posterior tibial vessels. Any inflammation or swelling in the area will compress on these structures, most significantly the tibial nerve.

A 72-year-old woman is admitted to the emergency department after an episode of stroke. During neurologic examination the patient shows no response to the ankle reflex test. Which of the following nerve roots is responsible for this reflex? A. L2 B. L3 C. L4 D. L5 E. S1

E. The ankle jerk reflex involves S1 and S2 levels. L2 to L4 are involved in the patellar reflex. L5 is not a component of a deep tendon reflex.

Lower limb angiography of an 82-year-old woman reveals a possible cause for her limb pain during her workout routines in the health spa. The artery that was occluded is one that should have been demonstrable passing between the proximal part of the space between the tibia and fibula. Which of the following arteries is most likely affected? A. Deep femoral B. Popliteal C. Posterior tibial D. Fibular (peroneal) E. Anterior tibial

E. The anterior tibial artery passes between the tibia and fibula proximally in the posterior compartment of the leg, whereas the posterior tibial artery continues in the posterior compartment of the leg, to its division into medial and lateral plantar arteries

A 45-year-old man is treated at the hospital after he fell from his bicycle. Radiologic examination reveals fractures both of the tibia and the fibula. On physical examination the patient has a foot drop, but normal eversion. Which of the following nerves is most likely injured? A. Tibial B. Common fibular (peroneal) C. Superficial fibular (peroneal) D. Saphenous E. Deep fibular (peroneal)

E. The deep fibular (peroneal) nerve is responsible for innervating the muscles of the anterior compartment of the leg, which are responsible for toe extension, foot dorsiflexion, and inversion. Injury to this nerve will result in foot drop and also loss of sensation between the first and second toes. (Damage to deep fibular nerve= foot drop = no dorsiflexion is possible)

A 58-year-old diabetic patient is admitted to the hospital with a painful foot. Physical examination reveals that the patient suffers from peripheral vascular disease. There is no detectable dorsalis pedis arterial pulse, but the posterior tibial pulse is strong. Which of the following arteries will most likely provide adequate collateral supply from the plantar surface to the toes and dorsum of the foot? A. Anterior tibial B. Fibular (peroneal) C. Arcuate D. Medial plantar E. Lateral plantar

E. The lateral plantar artery provides origin to the deep plantar arterial arch. Medially, the vascular arch anastomoses with the distal portion of the dorsalis pedis by way of the deep plantar artery. The anterior tibial artery continues as the dorsalis pedis at the ankle joint & cannot be the answer.

A 30-year-old woman is admitted to the emergency department with complaints of pain to the anterior left thigh. While participating in a 100-meter race, she felt a sudden onset of pain in the anterior midthigh area and could only limp to the finish line. Physical examination revealed a swollen, tender right thigh anteriorly. Extension of the knee was limited due to pain. Ultrasonography of the area revealed a defect in the fibers of the quadriceps muscle, confirmed by CT scan of the limb. Which of the following is the embryologic origin of the affected structure? A. Lateral plate mesoderm B. Dorsolateral migration of neural crest cells C. Preceded the development of chondrification centers D. Intermediate mesoderm E. Migration of cells from paraxial mesoderm

E. The muscles of the limbs develop from the myotome component of the somites. The somites are condensations of paraxial mesoderm that form after the formation of the trilaminar disc. The lateral plate mesoderm gives rise to the serous membranes, coverings of organs and the heart. The intermediate mesoderm gives rise to the urogenital system and its accessory glands. The chondrification centers precede the formation of the muscles, as it gives the skeletal framework. Neural crest cells give rise to dorsal root ganglia, leptomeninges, Schwann cells, sympathetic ganglia, and the chromaffin cells of the adrenal medulla. [LOOK AT EMBRYOLOGY & POSSIBLY SKIP!!!]

A 30-year-old man who is a bodybuilder presents to the physician's office complaining of pain and tingling sensation radiating down the inside of his thigh that was exacerbated upon thigh movement. A hernia through which opening would most likely cause this presentation? A. Femoral ring B. Superficial inguinal ring C. Deep inguinal ring D. Fossa ovalis E. Obturator canal

E. The obturator membrane is a fibrous sheet that almost completely closes the obturator foramen, leaving a small gap, the obturator canal, for the passage of the obturator nerve and vessels as they leave the pelvis to enter the medial thigh. The femoral canal is the small medial compartment for the lymph vessels

A 30-year-old woman is admitted to the emergency department with complaints of pain to the anterior left thigh. While participating in a 100-meter race, she felt a sudden onset of pain in the anterior midthigh area and could only limp to the finish line. Physical examination revealed a swollen, tender right thigh anteriorly. Extension of the knee was limited due to pain. Ultrasonography of the area revealed a defect in the fibers of the quadriceps muscle, confirmed by CT scan of the limb. Which of the following is the embryologic origin of the affected structure? A. Lateral plate mesoderm B. Dorsolateral migration of neural crest cells C. Preceded the development of chondrification centers D. Intermediate mesoderm E. Migration of cells from paraxial mesoderm

E. The paraxial mesoderm develops into somites. Limb muscles develop from the ventral myotome of the somites in response to molecular signals. Embryological derivatives of the lateral plate mesoderm include the circulatory and gut wall, body wall lining, and dermis. Derivative of the neural crest cells does not include the limb muscles. Chondrification is associated with cartilage formation and not muscles. The intermediate mesoderm eventually thins out laterally and becomes the mesoderm, which gives the circulatory and gut walls, plus the lining of the body wall and dermis. [LOOK AT EMBRYOLOGY & POSSIBLY SKIP!!!]

A 45-year-old man presents at the local emergency clinic with the complaint of a painful knee and difficulty in walking. A computed tomography (CT) scan examination reveals a very large cyst in the popliteal fossa compressing the tibial nerve. Which movement will most likely be affected? A. Dorsiflexion of the foot B. Flexion of the thigh C. Extension of the digits D. Extension of the leg E. Plantar flexion of the foot

E. The tibial nerve is responsible for innervating the posterior compartment of the leg. These muscles are responsible for knee flexion, plantar flexion, and intrinsic muscle functions of the foot. Compression of this nerve can affect plantar flexion of the foot.

A 43-year-old man visits the outpatient clinic with a painful, swollen knee joint. The patient's history reveals chronic gonococcal arthritis. A knee aspiration is ordered for bacterial culture of the synovial fluid. A standard suprapatellar approach is used, and the needle passes from the lateral aspect of the thigh into the region immediately proximal to and deep to the patella. Through which of the following muscles would the needle pass? A. Adductor magnus B. Short head of biceps femoris C. Rectus femoris D. Sartorius E. Vastus lateralis

E. The vastus lateralis muscle is located on the lateral aspect of the thigh. The distal portion of this muscle lies superficial to the proximal part of the lateral aspect of the joint capsule of the knee. When a needle is inserted superiorly and laterally to the patella, it penetrates the vastus lateralis muscle on its course to the internal capsule.

The semitendinosus, semimembranosus, and biceps femoris muscles (the hamstrings):

Extend the thigh and flex the leg.

Forced eversion vs. inversion of the foot:

Forced eversion of the foot avulses the medial malleolus or ruptures the deltoid ligament, whereas forced inversion avulses the lateral malleolus or tears the lateral collateral (anterior and posterior talofibular and calcaneofibular) ligament

Posterior cruciate ligament

Important because it prevents forward displacement of the femur on the tibia when the knee is flexed. The anterior cruciate ligament prevents backward displacement of the femur on the tibia.

The quadriceps femoris muscle:

Includes the rectus femoris muscle and the vastus medialis, intermedialis, and lateralis muscles. They extend the leg at the knee joint.

The deep peroneal and tibial nerves:

Innervate the chief evertors of the foot, which are the tibialis anterior, tibialis posterior, triceps surae, and extensor hallucis longus muscles. The tibialis anterior and extensor hallucis longus muscles are innervated by the deep peroneal nerve, and the tibialis posterior and triceps surae are innervated by the tibial nerve.

Sciatic Nerve (L4-S3)

Innervates the hamstring muscles by its tibial division, except for the short head of the biceps femoris, which is innervated by its common peroneal division.

Sural nerve

Innervates the skin on the lateral side of the foot

Superior gluteal nerve

Innervates the tensor fasciae latae

Lateral plantar nerve (similar to ulnar nerve)

Innervates: 1) quadratus plantae 2) muscles of the little toe 3) adductor hallucis 4) lumbricals 2 to 4 5) All of the interossei. It is sensory to the lateral side of the sole and the lateral three and a half digits.

Damage to the tibial nerve causes:

Loss of plantar flexion of the foot and impaired inversion resulting from paralysis of the tibialis posterior and causes a difficulty in getting the heel off the ground and a shuffling of the gait. It results in a characteristic clawing of the toes and sensory loss on the sole of the foot, affecting posture and locomotion.

A dislocated knee or fractured distal femur:

May injure the popliteal artery because of its deep position adjacent to the femur and the knee joint capsule

Damage to the common peroneal (fibular) nerve:

May occur as a result of fracture of the head or neck of the fibula because it passes behind the head of the fibula and then winds laterally around the neck of the fibula. The nerve damage results in foot drop (loss of dorsiflexion) and loss of sensation on the dorsum of the foot and lateral aspect of the leg and causes paralysis of all muscles in the anterior and lateral compartments of the leg (dorsiflexor and evertor muscles of the foot).

Unhappy triad or O'Donoghue triad of the knee joint

May occur when a football player's cleated shoe is planted firmly in the turf and the knee is struck from the lateral side. It is indicated by a markedly swollen knee and results in tenderness on application of pressure along the tibial collateral ligament. It is characterized by the: (a) rupture of the tibial collateral ligament, as a result of excessive abduction; (b) tearing of the anterior cruciate ligament, as a result of forward displacement of the tibia; and (c) injury to the medial meniscus, as a result of the tibial collateral ligament attachment. However, lateral meniscus injuries are commonly seen among athletes.

Medial (central or intrapelvic) dislocation of the hip joint

Occurs through a medial tearing of the joint capsule, and the dislocated femoral head lies medial to the pubic bone. This may be accompanied by acetabular fracture and rupture of the bladder.

Posterior dislocation of the hip joint

Occurs through a posterior tearing of the joint capsule, accounts for approximately 90% of hip dislocations, and the fractured femoral head lies posterior to the acetabulum or the ischium, as occurs in a head-on collision. It results in a probable rupture of both the posterior acetabular labrum and the ligamentum capitis femoris and usually in the injury of the sciatic nerve. *It results in the affected lower limb being shortened, flexed, adducted, and medially rotated.*

Knee-jerk (patellar) reflex:

Occurs when the patellar ligament is tapped, resulting in a sudden contraction of the quadriceps femoris. It tests the L2 to L4 spinal (femoral) nerves by activating muscle spindle in the quadriceps. Both afferent and efferent impulses are transmitted in the femoral nerve.

Different forms of bursitis:

Prepatellar bursitis (housemaid knee) is inflammation and swelling of the prepatellar bursa. Infrapatellar (superficial) bursitis (clergyman knee) is inflammation of the infrapatellar bursa located between the patellar ligament and the skin (the deep bursa lies between the patellar ligament and the tibia). Popliteal (Baker) cyst is a swelling behind the knee, caused by knee arthritis, meniscus injury, or herniation or tear of the joint capsule. It impairs flexion and extension of the knee joint, and the pain gets worse when the knee is fully extended, such as during prolonged standing or walking. It can be treated by draining and decompressing the cyst.

Attachments of the AIIS

Rectus femoris

Transverse patellar fracture:

Results from a blow to the knee or from sudden contraction of the quadriceps muscle. The proximal fragment of the patella is pulled superiorly with the quadriceps tendon, and the distal fragment remains with the patellar ligament.

Fracture of the neck of the femur:

Results in ischemic necrosis of the neck and head because of an interruption of blood supply from the root of the femoral neck to the femoral head by the medial femoral circumflex artery, except for its small proximal part. It causes a pull of the distal fragment upward by the quadriceps femoris, adductors, and hamstring muscles so that the affected lower limb is shortened with lateral rotation.

Positive Trendelenburg sign

Seen in a fracture of the femoral neck, dislocated hip joint (head of femur), or weakness and paralysis of the gluteus medius (abductor). If the right gluteus medius muscle is paralyzed, the left side (sound side) of the pelvis falls (sags) instead of rising; normally, the pelvis rises.

(BASED ON PREVIOUS) After injury to this nerve, which of the following muscles could be paralyzed? (A) Gastrocnemius (B) Popliteus (C) Extensor hallucis longus (D) Flexor digitorum longus (E) Tibialis posterior

The Answer is C. The extensor hallucis longus is innervated by the deep peroneal nerve, whereas other muscles are innervated by the posterior tibial nerve.

While playing football, a 19-year-old college student receives a twisting injury to his knee when being tackled from the lateral side. Which of the following conditions most likely has occurred? (A) Tear of the medial meniscus (B) Ruptured fibular collateral ligament (C) Tenderness on pressure along the fibular collateral ligament (D) Injury of the posterior cruciate ligament (E) Swelling on the back of the knee joint

The Answer is A. The "unhappy triad" of the knee joint is characterized by tear of the medial meniscus, rupture of the tibial collateral ligament, and rupture of the anterior cruciate ligament. This injury may occur when a cleated shoe, as worn by football players, is planted firmly in the turf and the knee is struck from the lateral side. Tenderness along the medial collateral ligament and over the medial meniscus and swelling on the front of the joint are due to excessive production of synovial fluid, which fills the joint cavity and the suprapatellar bursa

An orthopedic surgeon ligates the posterior tibial artery at its origin. Which of the following arteries has no blood flow immediately after the ligation? (A) Peroneal (B) Dorsalis pedis (C) Superior medial genicular (D) Anterior tibial (E) Descending genicular

The Answer is A. The peroneal artery is a branch of the posterior tibial artery. The dorsalis pedis artery begins anterior to the ankle as the continuation of the anterior tibial artery. The superior medial genicular artery is a branch of the popliteal artery, and the descending genicular artery arises from the femoral artery.

A motorcyclist falls from his bike in an accident and gets a deep gash that severs the superficial peroneal nerve near its origin. Which of the following muscles is paralyzed? (A) Peroneus longus (B) Extensor hallucis longus (C) Extensor digitorum longus (D) Peroneus tertius (E) Extensor digitorum brevis

The Answer is A. The superficial peroneal nerve supplies the peroneus longus and brevis muscles. Other muscles are innervated by the deep peroneal nerve.

A patient experiences paralysis of the muscle that originates from the femur and contributes directly to the stability of the knee joint. Which of the following muscles is involved? (A) Vastus lateralis (B) Semimembranosus (C) Sartorius (D) Biceps femoris (long head) (E) Rectus femoris

The Answer is A. The vastus lateralis muscles arise from the femur and all the other muscles originate from the hip (coxal) bone. The biceps femoris inserts on the fibula, and other muscles insert on the tibia; thus, all of them contribute to the stability of the knee joint. Remember that the vastus lateralis is a quadriceps femoris muscle.

A 10-year-old boy falls from a tree house. The resultant heavy compression of the sole of his foot against the ground caused a fracture of the head of the talus. Which of the following structures is unable to function normally? (A) Transverse arch (B) Medial longitudinal arch (C) Lateral longitudinal arch (D) Tendon of the peroneus longus (E) Long plantar ligament

The Answer is B. *The keystone of the medial longitudinal arch of the foot is the head of the talus* which is located at the summit between the sustentaculum tali and the navicular bone. The medial longitudinal arch is supported by the spring ligament and the tendon of the flexor hallucis longus muscle. The cuboid bone serves as the keystone of the lateral longitudinal arch, which is supported by the peroneus longus tendon and the long and short plantar ligaments. The transverse arch is formed by the navicular, the three cuneiform, the cuboid, and the five metatarsal bones and is supported by the peroneus longus tendon and the transverse head of the adductor hallucis.

A 27-year-old patient exhibits a loss of skin sensation and paralysis of muscles on the plantar aspect of the medial side of the foot. Which of the following nerves is most likely damaged? (A) Common peroneal (B) Tibial (C) Superficial peroneal (D) Deep peroneal (E) Sural

The Answer is B. The common peroneal nerve divides into the deep peroneal nerve, which innervates the anterior muscles of the leg and supplies the adjacent skin of the first and second toes, and the superficial peroneal nerve, which innervates the lateral muscles of the leg and supplies the skin on the side of the lower leg and the dorsum of the ankle and foot. The sural nerve supplies the lateral aspect of the foot and the little toe. HELP!!!!

A 47-year-old woman is unable to invert her foot after she stumbled on her driveway. Which of the following nerves are most likely injured? (A) Superficial and deep peroneal (B) Deep peroneal and tibial (C) Superficial peroneal and tibial (D) Medial and lateral plantar (E) Obturator and tibial

The Answer is B. The deep peroneal and tibial nerves innervate the chief evertors of the foot, which are the tibialis anterior, tibialis posterior, triceps surae, and extensor hallucis longus muscles. The tibialis anterior and extensor hallucis longus muscles are innervated by the deep.

A 32-year-old carpenter fell from the roof. The lateral longitudinal arch of his foot was flattened from fracture and displacement of the keystone for the arch. Which of the following bones is damaged? (A) Calcaneus (B) Cuboid bone (C) Head of the talus (D) Medial cuneiform (E) Navicular bone

The Answer is B. The keystone for the lateral longitudinal arch is the cuboid bone, whereas the keystone for the medial longitudinal arch is the head of the talus. The calcaneus, navicular, and medial cuneiform bones form a part of the medial longitudinal arch, but they are not keystones. The calcaneus also forms a part of the lateral longitudinal arch.

If the lateral (fibular) collateral ligament is torn by this fracture, which of the following conditions may occur? (A) Abnormal passive abduction of the extended leg (B) Abnormal passive adduction of the extended leg (C) Anterior displacement of the femur on the tibia (D) Posterior displacement of the femur on the tibia (E) Maximal flexion of the leg

The Answer is B. The lateral (fibular) collateral ligament prevents adduction at the knee. Therefore, a torn lateral collateral ligament can be recognized by abnormal passive adduction of the extended leg. Abnormal passive abduction of the extended leg may occur when the medial (tibial) collateral ligament is torn. The anterior cruciate ligament prevents posterior displacement of the femur on the tibia; the posterior cruciate ligament prevents anterior displacement of the femur on the tibia. In addition, the posterior cruciate ligament is taut when the knee is fully flexed.

Which of the following arteries could be damaged by fracture of the head and neck of the fibula? (A) Popliteal (B) Posterior tibial (C) Anterior tibial (D) Peroneal (E) Lateral inferior genicular

The Answer is C. *The anterior tibial artery, which arises from the popliteal artery, enters the anterior compartment* by passing through the gap between the fibula and tibia at the upper end of the interosseous membrane. The other arteries would not be affected because they are not closely associated with the head and neck of the fibula.

A 34-year-old woman sustains a deep cut on the dorsum of the foot just distal to her ankle joint by a falling kitchen knife. A physician in the emergency department has ligated the dorsalis pedis artery proximal to the injured area. Which of the following conditions most likely occurs as a result of the injury? (A) Ischemia in the peroneus longus muscle (B) Aneurysm in the plantar arterial arch (C) Reduction of blood flow in the medial tarsal artery (D) Low blood pressure in the anterior tibial artery (E) High blood pressure in the arcuate artery

The Answer is C. *Reduction of blood flow in the medial tarsal artery occurs because it is a branch of the dorsalis pedis artery, which begins at the ankle joint as the continuation of the anterior tibial artery.* The anterior tibial and peroneal arteries supply the peroneus longus muscle. The deep plantar arterial arch is formed mainly by the lateral plantar artery. Blood pressure in the anterior tibial artery should be higher than normal. The arcuate artery should have a low blood pressure because it is a terminal branch of the dorsalis pedis artery.

A basketball player was hit in the thigh by an opponent's knee. Which of the following arteries is likely to compress and cause ischemia because of the bruise and damage to the extensor muscles of the leg? (A) Popliteal (B) Deep femoral (C) Anterior tibial (D) Posterior tibial (E) Peroneal

The Answer is C. A muscular spasm or hypertrophy of the extensor muscles of the leg may compress the anterior tibial artery, causing ischemia. The popliteal artery supplies muscles of the popliteal fossa. The deep femoral artery supplies deep muscles of the thigh. The posterior tibial and peroneal (fibular) arteries supply muscles of the posterior and lateral compartments of the leg.

If the acetabulum is fractured at its posterosuperior margin by dislocation of the hip joint, which of the following bones could be involved? (A) Pubis (B) Ischium (C) Ilium (D) Sacrum (E) Head of the femur

The Answer is C. The acetabulum is a cup-shaped cavity on the lateral side of the hip bone and is formed superiorly by the ilium, posteroinferiorly by the ischium, and anteromedially by the pubis.

A 17-year-old boy was stabbed during a gang fight, resulting in the transection of the obturator nerve. Which of the following muscles is completely paralyzed? (A) Pectineus (B) Adductor magnus (C) Adductor longus (D) Biceps femoris (E) Semimembranosus

The Answer is C. The adductor longus is innervated by only the obturator nerve. Thus, injury here could completely paralyze the adductor longus. The pectineus is innervated by both the obturator and femoral nerves. The adductor magnus is innervated by both the obturator nerve and tibial part of the sciatic nerve. The biceps femoris is innervated by the tibial portion (long head) and common peroneal portion (short head) of the sciatic nerve. The semimembranosus is innervated by the tibial portion of the sciatic nerve.

A 20-year-old college student receives a severe blow on the inferolateral side of the left knee joint while playing football. Radiographic examination reveals a fracture of the head and neck of the fibula. Which of the following nerves is damaged? (A) Sciatic (B) Tibial (C) Common peroneal (D) Deep peroneal (E) Superficial peroneal

The Answer is C. The common peroneal nerve is vulnerable to injury as it passes behind the head of the fibula and then winds around the neck of the fibula and pierces the peroneus longus muscle, where it divides into the deep and superficial peroneal nerves. In addition, the deep and superficial peroneal nerves pass superficial to the neck of the fibula in the substance of the peroneus longus muscle and are less susceptible to injury than the common peroneal nerve. Other nerves are not closely associated with the head and neck of the fibula.

A patient presents with sensory loss on adjacent sides of the great and second toes and impaired dorsiflexion of the foot. These signs probably indicate damage to which of the following nerves? (A) Superficial peroneal (B) Lateral plantar (C) Deep peroneal (D) Sural (E) Tibial

The Answer is C. The deep peroneal nerve supplies the anterior muscles of the leg, including the tibialis anterior, extensor hallucis longus, extensor digitorum longus, and peroneus tertius muscles, which dorsiflex the foot. The medial branch of the deep peroneal nerve supplies the skin of adjacent sides of the great and second toes, whereas the lateral branch supplies the extensor digitorum brevis and extensor hallucis brevis. The superficial peroneal nerve innervates the peroneus longus and brevis, which plantar flexes the foot, and supplies the skin on the side of the lower leg and the dorsum of the ankle and foot. The tibial nerve innervates the muscles of the posterior compartment that plantar flexes and supplies the skin on the heel and plantar aspect of the foot. The lateral plantar nerve innervates muscles and skin of the lateral plantar aspect of the foot. The sural nerve supplies the skin on the posterolateral aspect of the leg and the lateral aspect of the foot and the little toe.

A 21-year-old man was involved in a motorcycle accident, resulting in destruction of the groove in the lower surface of the cuboid bone. Which of the following muscle tendons is most likely damaged? (A) Flexor hallucis longus (B) Peroneus brevis (C) Peroneus longus (D) Tibialis anterior (E) Tibialis posterior

The Answer is C. The groove in the lower surface of the cuboid bone is occupied by the tendon of the peroneus longus muscle. The flexor hallucis longus tendon occupies a groove on the posterior surface of the body of the talus and a groove on the inferior surface of the calcaneus during its course. The tibialis posterior muscle tendon occupies the medial malleolar groove of the tibia. Other muscle tendons are not in the groove of the tarsal bones.

Rupture of the ligamentum teres capitis femoris may lead to damage to a branch of which of the following arteries? (A) Medial circumflex femoral (B) Lateral circumflex femoral (C) Obturator (D) Superior gluteal (E) Inferior gluteal

The Answer is C. The obturator artery gives rise to an acetabular branch that runs in the round ligament of the head of the femur.

A 22-year-old patient is unable to "unlock" the knee joint to permit flexion of the leg. Which of the following muscles is most likely damaged? (A) Rectus femoris (B) Semimembranosus (C) Popliteus (D) Gastrocnemius (E) Biceps femoris

The Answer is C. The popliteus muscle rotates the femur laterally ("unlocks" the knee) or rotates the tibia medially, depending on which bone is fixed. This action results in unlocking of the knee joint to initiate flexion of the leg at the joint. The rectus femoris flexes the thigh and extends the knee. The gastrocnemius flexes the knee and plantar flexes the foot. The semimembranosus extends the thigh and flexes and rotates the leg medially. The biceps femoris extends the thigh and flexes and rotates the leg laterally.

A 24-year-old woman presents to her physician with weakness in flexing the hip joint and extending the knee joint. Which muscle is most likely involved in this scenario? (A) Sartorius (B) Gracilis (C) Rectus femoris (D) Vastus medialis (E) Semimembranosus

The Answer is C. The rectus femoris flexes the thigh and extends the leg. The sartorius can flex both the hip and knee joints. The gracilis adducts and flexes the thigh and flexes the leg, the vastus medialis extends the knee joint, and the semimembranosus extends the hip joint and flexes the knee joint

The woman undergoes hip surgery. If all of the arteries that are part of the cruciate anastomosis of the upper thigh are ligated, which of the following arteries maintains blood flow? (A) Medial femoral circumflex (B) Lateral femoral circumflex (C) Superior gluteal (D) Inferior gluteal (E) First perforating

The Answer is C. The superior gluteal artery does not participate in the cruciate anastomosis of the thigh. The inferior gluteal artery, transverse branches of the medial and lateral femoral circumflex arteries, and an ascending branch of the first perforating artery form the cruciate anastomosis of the thigh.

A patient experiences weakness in dorsiflexing and inverting the foot. Which of the following muscles is damaged? (A) Peroneus longus (B) Peroneus brevis (C) Tibialis anterior (D) Extensor digitorum longus (E) Peroneus tertius

The Answer is C. The tibialis anterior can dorsiflex and invert the foot. The peroneus longus and brevis muscles can plantar flex and evert the foot, the peroneus tertius can dorsiflex and evert the foot, and the extensor digitorum longus can dorsiflex the foot and extend the toes.

A construction worker is hit on the leg with a concrete block and is subsequently unable to plantar flex and invert his foot. Which of the following muscles is most likely damaged? (A) Extensor digitorum longus (B) Tibialis anterior (C) Tibialis posterior (D) Peroneus longus (E) Peroneus brevis

The Answer is C. The tibialis posterior can plantar flex and invert the foot. The extensor digitorum longus can dorsiflex and evert the foot The tibialis anterior can dorsiflex and invert the foot The peroneus longus (also known as fibularis longus) and brevis can plantar flex and evert the foot.

The obturator nerve and the sciatic (tibial portion) nerve of a 15-year-old boy are transected as a result of a motorcycle accident. This injury would result in complete paralysis of which of the following muscles? (A) Rectus femoris (B) Biceps femoris, short head (C) Pectineus (D) Adductor magnus (E) Sartorius

The Answer is D. *The adductor magnus is innervated by both the obturator and sciatic (tibial portion) nerves.* Hence, a lesion here could cause paralysis. The rectus femoris and sartorius are innervated by the femoral nerve. The biceps femoris long head is innervated by the tibial portion of the sciatic nerve, whereas the short head is innervated by the common peroneal portion of the sciatic nerve. The pectineus is innervated by both the femoral and obturator nerves.

A thoracic surgeon is going to collect a portion of the greater saphenous vein for coronary bypass surgery. He has observed that this vein runs (A) Posterior to the medial malleolus (B) Into the popliteal vein (C) Anterior to the medial condyles of the tibia and femur (D) Superficial to the fascia lata of the thigh (E) Along with the femoral artery

The Answer is D. *The greater saphenous vein ascends superficial to the fascia lata. It courses anterior to the medial malleolus* and posterior to the medial condyles of the tibia and femur and terminates in the femoral vein by passing through the saphenous opening. The small saphenous vein drains into the popliteal vein. The greater saphenous vein does not run along with the femoral artery.

Which of the following conditions would occur from fracture of the head and neck of the fibula? (A) Ischemia in the gastrocnemius (B) Loss of plantar flexion (C) Trendelenburg sign (D) Anterior tibial compartment syndrome (E) Flat foot

The Answer is D. Anterior tibial compartment syndrome is characterized by ischemic necrosis of the muscles of the anterior tibial compartment of the leg resulting from damage to the anterior tibial artery. The gastrocnemius receives blood from sural branches of the popliteal artery. Loss of plantar flexion is due to necrosis of the posterior muscles of the leg, which are supplied by the posterior tibial and peroneal arteries. Trendelenburg sign is caused by weakness or paralysis of the gluteus medius and minimus muscles. Flat foot results from the collapse of the medial longitudinal arch of the foot.

A patient with hereditary blood clotting problems presents with pain in the back of her knee. An arteriograph reveals a blood clot in the popliteal artery at its proximal end. Which of the following arteries will allow blood to reach the foot? (A) Anterior tibial (B) Posterior tibial (C) Peroneal (D) Lateral circumflex femoral (E) Superior medial genicular

The Answer is D. If the proximal end of the popliteal artery is blocked, blood may reach the foot by way of the descending branch of the lateral circumflex femoral artery, which participates in the anastomosis around the knee joint. Other blood vessels are direct or indirect branches of the popliteal artery. Remember that the popliteal artery is a deeply placed continuation of the femoral artery!!!!! So if the popliteal is blocked then choose a branch of the femoral as the answer.

Fracture of the neck of the femur results in avascular necrosis of the femoral head, probably resulting from lack of blood supply from which of the following arteries? (A) Obturator (B) Superior gluteal (C) Inferior gluteal (D) Medial femoral circumflex (E) Lateral femoral circumflex

The Answer is D. In adults, the chief arterial supply to the head of the femur is from the branches of the medial femoral circumflex artery. The lateral femoral circumflex artery may supply the femoral head by anastomosing with the medial femoral circumflex artery. The posterior branch of the obturator artery gives rise to the artery of the head of the femur, which runs in the round ligament of the femoral head and is usually insufficient to supply the head of the femur in adults but is an important source of blood to the femoral head in children. The superior and inferior gluteal arteries do not supply the head of the femur.

A 24-year-old woman complains of weakness when she extends her thigh and rotates it laterally. Which of the following muscles is paralyzed? (A) Obturator externus (B) Sartorius (C) Tensor fasciae latae (D) Gluteus maximus (E) Semitendinosus

The Answer is D. The gluteus maximus can extend and rotate the thigh laterally. The obturator externus rotates the thigh laterally. The sartorius can flex both the hip and knee joints. The tensor fasciae latae can flex and medially rotate the thigh. The semitendinosus can extend the thigh and medially rotate the leg.

An elderly woman fell at home and fractured the greater trochanter of her femur. Which of the following muscles would continue to function normally? (A) Piriformis (B) Obturator internus (C) Gluteus medius (D) Gluteus maximus (E) Gluteus minimus

The Answer is D. The gluteus maximus is inserted into the gluteal tuberosity of the femur and the iliotibial tract. All of the other muscles insert on the greater trochanter of the femur, and their functions are impaired.

A 52-year-old woman slipped and fell and now complains of being unable to extend her leg at the knee joint. Which of the following muscles was paralyzed as a result of this accident? (A) Semitendinosus (B) Sartorius (C) Gracilis (D) Quadriceps femoris (E) Biceps femoris

The Answer is D. The quadriceps femoris muscle includes the rectus femoris muscle and the vastus medialis, intermedialis, and lateralis muscles. They extend the leg at the knee joint. The semitendinosus, semimembranosus, and biceps femoris muscles (the hamstrings) extend the thigh and flex the leg. The sartorius and gracilis muscles can flex the thigh and the leg.

A construction worker falls feet first from a roof. He sustains a fracture of the groove on the undersurface of the sustentaculum tali of the calcaneus bone. Which of the following muscle tendons is most likely torn? (A) Flexor digitorum brevis (B) Flexor digitorum longus (C) Flexor hallucis brevis (D) Flexor hallucis longus (E) Tibialis posterior

The Answer is D. The tendon of the flexor hallucis longus muscle occupies first the groove on the posterior surface of the talus and then the groove on the undersurface of the sustentaculum tali. None of the other tendons would have been affected in such an injury.

The woman experiences weakness when abducting and medially rotating the thigh after this accident. Which of the following muscles is most likely damaged? (A) Piriformis (B) Obturator internus (C) Quadratus femoris (D) Gluteus maximus (E) Gluteus minimus

The Answer is E. *The gluteus medius or minimus abducts and rotates the thigh medially.* The piriformis, obturator internus, quadratus femoris, and gluteus maximus muscles can rotate the thigh laterally

A 20-year-old patient cannot flex and medially rotate the thigh while running and climbing. Which of the following muscles is most likely damaged? (A) Semimembranosus (B) Sartorius (C) Rectus femoris (D) Vastus intermedius (E) Tensor fasciae latae

The Answer is E. *The tensor fasciae latae can flex and medially rotate the thigh, so this is the muscle most likely damaged.* The hamstring muscles (semitendinosus, semimembranosus, and biceps femoris) can extend the thigh and flex the leg. The sartorius can flex the thigh and leg. The rectus femoris can flex the thigh and extend the leg. The vastus intermedius can extend the leg.

A knife wound penetrates the superficial vein that terminates in the popliteal vein. Bleeding occurs from which of the following vessels? (A) Posterior tibial vein (B) Anterior tibial vein (C) Peroneal vein (D) Great saphenous vein (E) Lesser saphenous vein

The Answer is E. The lesser (small) saphenous vein ascends on the back of the leg in company with the sural nerve and terminates in the popliteal vein. The peroneal vein empties into the posterior tibial vein. The anterior and posterior tibial veins are deep veins and join to form the popliteal vein. The great saphenous vein drains into the femoral vein.

A patient is involved in a motorcycle wreck that results in avulsion of the skin over the anterolateral leg and ankle. Which of the following structures is most likely destroyed with this type of injury? (A) Deep peroneal nerve (B) Extensor digitorum longus muscle tendon (C) Dorsalis pedis artery (D) Great saphenous vein (E) Superficial peroneal nerve

The Answer is E. The superficial peroneal nerve emerges between the peroneus longus and peroneus brevis muscles and descends superficial to the extensor retinaculum of the ankle on the anterolateral side of the leg and ankle, innervating the skin of the lower leg and foot. The great saphenous vein begins at the medial end of the dorsal venous arch of the foot and ascends in front of the medial malleolus and along the medial side of the tibia along with the saphenous nerve. Other structures pass deep to the extensor retinaculum.

KNOW THIS:

The gracilis is a relatively weak member of the adductor group of muscles, and thus surgeons often transplant this muscle or part of it, with nerve and blood vessels, to replace a damaged muscle in the hand. The proximal muscle attachments are in the inguinal region or groin. Muscle strains of the adductor longus may occur in horseback riders and produce pain because the riders adduct their thighs to keep from falling from the animal.

KNOW THIS:

The great saphenous nerve remains intact because it is not in the adductor canal. The adductor canal contains the femoral vessels, the saphenous nerve, and the nerve to the vastus medialis.

A 15-year-old girl is struck by a car while crossing the street. She suffers numerous pelvic injuries, including tearing of the sacrotuberous ligament. The damage to this ligament will most likely cause direct trauma to which of the following muscles? (A) Gluteus maximus (B) Gluteus medius (C) Gluteus minimus (D) Gemelli (E) Obturator externus

The answer is A: Gluteus maximus. The sacrotuberous ligament runs from the lower lateral sacrum and the coccyx to the ischial tuberosity. This ligament is one of the originations of the gluteus maximus.

A 16-year-old boy was fishing barefoot in a muddy river when the plantar surface of his foot was cut by unseen debris. He suffers a large transverse cut, penetrating the first two layers of his plantar musculature, in the area of the first cuneiform bone. In the emergency room, his physician notes a complete inability to flex and abduct the big toe and numbness on the plantar aspect of the three medial toes. Which of the following nerves is most likely damaged? (A) Medial plantar nerve (B) Lateral plantar nerve (C) Sural nerve (D) Deep fibular nerve (E) Superficial fibular nerve

The answer is A: Medial plantar nerve. The medial plantar nerve, which is homologous to the median nerve in the hand, innervates four intrinsic foot muscles: first Lumbrical, Abductor hallucis, Flexor digitorum brevis, and Flexor hallucis brevis (mnemonic = "LAFF" muscles). This nerve supplies cutaneous innervation to the medial three and a half toes on the plantar surface of the foot.

A physician tests the calcaneal tendon (ankle jerk) reflex as shown. A normal response of plantar fl exion of the ankle joint is noted. This myotatic (deep tendon) reflex confirms the integrity of what nerve? (A) Tibial nerve (B) Deep fi bular nerve (C) Superfi cial fi bular nerve (D) Medial plantar nerve (E) Lateral plantar nerve

The answer is A: Tibial nerve. A positive response to the calcaneal (Achilles) tendon refl x causes plantar flexion of the ankle joint via the contractions of the gastrocnemius and soleus muscles, which insert distally into the calcaneal tendon. These muscles are innervated by the tibial nerve, and the plantar flexion of the ankle joint confirms the integrity of this nerve and the S1-2 spinal segments, from which this nerve is primarily derived. The tibial nerve is a terminal branch of the sciatic nerve that supplies the posterior compartment of the leg, including the superficial compartment where the gastrocnemius and soleus muscles reside The tibial nerve divides into the medial and lateral plantar nerves to innervate the muscles of the sole of the foot. The lateral plantar nerve, which is homologous to the ulnar nerve in the hand, innervates all of the intrinsic foot muscles, with the exception of the four "LAFF" muscles supplied by the medial plantar nerve. Because this nerve does not supply muscles that cross the ankle joint, it cannot be responsible for the plantar flexion

A roofing installer falls off a high ladder and lands with the sole of his right foot hitting the ground first. He suffers a fracture and inferior displacement of the sustentaculum tali of the calcaneus. Which of the following structures is most likely torn? (A) Tendon of the tibialis posterior muscle (B) Tendon of the flexor hallucis longus (FHL) muscle (C) Tendons of the fl exor digitorum brevis muscle (D) Small saphenous vein (E) Plantar arterial arch

The answer is B: *Tendon of the flexor hallucis longus (FHL) muscle. The sustentaculum tali is a bony ledge projecting off the medial side of the calcaneus* It supports the head of the talus and serves as a pulley-like device for the tendon of the FHL. The plantar arterial arch is located deep in the plantar aspect of the foot, crossing the metatarsals. It is formed from the lateral plantar artery and the deep plantar artery (a branch of the dorsalis pedis artery).

A 21-year-old female basketball player lands on her opponent's foot after jumping to rebound the basketball. Her foot is forcefully inverted, and when leaving the court, she tells her trainer that she twisted or sprained her ankle. After getting her ankle taped for support, she reenters the game. What ligament was most likely damaged? (A) Calcaneofibular ligament (B) Anterior talofibular ligament (C) Posterior talofibular ligament (D) Plantar calcaneonavicular ligament (E) Medial (deltoid) ligament

The answer is B: Anterior talofibular ligament. The ankle is the most frequently injured major joint in the body. Torn ligaments (ankle sprains) are commonly seen following inversion injuries, where the joint is twisted on a weight-bearing plantar flexed foot (as described in this patient). The most commonly sprained ankle ligament is the anterior talofibular ligament on the lateral side of the ankle, and this injury results in instability of the joint.

As part of a physical examination to evaluate lower limb function, a physician places her hands on the dorsum of the patient's foot and asks the patient to dorsiflex the ankle joint against resistance, as shown. What nerve is the doctor testing? (A) Tibial nerve (B) Deep fibular nerve (C) Superficial fibular nerve (D) Medial plantar nerve (E) Lateral plantar nerve

The answer is B: Deep fibular nerve. The deep fibular (peroneal) nerve is a terminal branch of the common fibular that supplies motor innervation to the four muscles of the anterior compartment of the leg: (1) tibialis anterior (2) extensor digitorum longus (3) extensor hallucis longus (4) fibularis (peroneus) tertius *These four muscles are responsible for dorsiflexion of the ankle.* The deep fibular nerve also innervates the extensor digitorum brevis and extensor hallucis brevis, which are intrinsic muscles of the foot, sends articular branches to joints it crosses, and supplies cutaneous innervation to the first interdigital cleft.

The relative positions of blood vessels can sometimes be explained in terms of rotation of the limbs during development. For example, upon entering the femoral triangle the femoral artery resides ____ to the femoral vein, whereas in the popliteal fossa the popliteal artery lies ____ to the popliteal vein. (A) medial...posterior (B) lateral...deep (C) lateral...posterior (D) medial...superficial (E) anterior...lateral

The answer is B: lateral...deep.

A patient presents with extreme pain due to arterial insufficiency in the posterior femoral compartment. This compartment of the thigh receives its blood supply mainly from the perforating arteries. An arteriogram confirms partial occlusion of the artery that gives rise to these perforating arteries. What artery is occluded in the arteriogram? (A) Femoral artery (B) Profunda femoris artery (C) Obturator artery (D) Popliteal artery (E) Medial femoral circumflex artery

The answer is B: Profunda femoris artery. The profunda femoris artery (deep femoral artery; deep artery of the thigh) gives off the perforating arteries that supply blood to the posterior femoral compartment. Arising from the femoral artery, the profunda femoris artery gives off the medial and lateral circumflex femoral arteries before it descends to give off a series of (usually four in number) perforating arteries. These arteries are so named because they pierce (perforate) through the adductor magnus muscle to reach the posterior compartment of the thigh. The perforating arteries supply the adductor magnus and hamstring muscles of the posterior compartment of the thigh.

After spending 2 days moving heavy furniture out of her house, a 56-year-old woman goes to an emergency room in acute pain. The patient reports nausea, vomiting, and severe abdominal pain. On examination, the doctor discovers a painful, globular mass located inferior and lateral to the pubic tubercle. Given her presentation and history, what is the most likely diagnosis? (A) Indirect inguinal hernia (B) Direct inguinal hernia (C) Femoral hernia (D) Lymphadenitis of superficial inguinal nodes (E) Fractured hip

The answer is C: Femoral hernia. The femoral ring is a weakened aspect in the anterior abdominal wall through which a femoral hernia enters the femoral canal. This type of hernia often contains abdominal viscera and is more common in females due to their wider pelves. Femoral herniae are also more susceptible to strangulation, in which case the blood supply can be interrupted due to the sharp boundaries of the femoral ring, particularly the lacunar ligament. Because this patient is female and the sudden onset of her symptoms following strenuous lifting, strangulation of a femoral hernia is the most likely diagnosis. Her nausea and vomiting may be due to a small bowel obstruction or may be a result of the severe pain due to the ischemia associated with the hernia.

A 23-year-old female medical student notices she was gaining weight due to a sedimentary lifestyle and compulsive studying. Therefore, she decides to run in a marathon and starts her training by running 6 miles each morning before class. After 2 weeks, she presents with right lateral knee pain and inflammation, specifically in the area of the lateral femoral epicondyle. This pain intensifies throughout her morning jogs, especially when her right foot strikes the ground or when she is running downhill. What is most likely diagnosis in this patient? (A) Sprained fi bular (lateral) collateral ligament (FCL) (B) Patellofemoral pain syndrome (C) Iliotibial band syndrome (ITBS) (D) Torn lateral meniscus (E) Pes anserinus bursitis

The answer is C: Iliotibial band syndrome (ITBS). ITBS is one of the leading causes of lateral knee pain in runners, and it is often seen in runners who increase the intensity of their workouts too rapidly. The iliotibial band (tract) is a lateral thickening of the fascia lata that extends inferiorly to the anterolateral aspect of the lateral condyle of the tibia and stabilizes the knee during running.

When standing upright, the femur moves into the locked position by slightly hyperextending, gliding posteriorly, and medially rotating on the tibial plateaus. Which of the following muscles acts to initiate the unlocking of the knee? (A) Biceps femoris (B) Gastrocnemius (C) Popliteus (D) Sartorius (E) Plantaris

The answer is C: Popliteus. As described in the question, holding the upright posture during standing includes locking the knee in a stable position. Locking is a complex event in which the femur slightly hyperextends, glides posterior, and rotates medially on the tibial condyles. Therefore, unlocking the knee (necessary in shifting from standing to locomoting) must call for the reverse movements. The popliteus lies deep and obliquely across the posterior aspect of the knee, from the lateral condyle of the femur to the posterior aspect of the proximal tibia. It unlocks the knee by initiating flexion and laterally rotating the femur on the tibia.

A man working in a junkyard trips and falls into a pile of scrap metal, suffering a deep cut immediately posterior to the lateral malleolus. Which of the following is most likely to be injured? (A) Saphenous nerve (B) Tendon of the fibularis (peroneus) tertius (C) Tendon of the fibularis (peroneus) longus (D) Posterior tibial artery (E) Great saphenous vein

The answer is C: Tendon of the fibularis (peroneus) longus. The tendons of the fibularis (peroneus) longus and brevis muscles pass tightly against the posterior side of the lateral malleolus as they leave the lateral compartment of the leg and cross the ankle. Therefore, both muscles act to produce plantar flexion of the foot and may be injured in trauma posterior to the lateral malleolus.

An overweight woman participates in her first rugby match without proper training and conditioning. Upon catching the opening kickoff, she awkwardly twists her right knee, screams in pain, and falls to the ground. The team manager notes her patella is dislocated, residing on the lateral side of her knee. After straightening the woman's knee, the patellar dislocation is reduced (goes back into place). To prevent future dislocation of the patella, what specific muscle should be targeted during rehabilitation? (A) Vastus intermedius (B) Vastus lateralis (C) Vastus medialis (D) Rectus femoris (E) Tibialis anterior

The answer is C: Vastus medialis. The vastus medialis muscle, a component of the quadriceps femoris, inserts into the patella and the tibial tuberosity through the common quadriceps tendon and patellar ligament. This muscle helps extend the leg at the knee joint, but it also maintains a medial pull on the patella, reducing the lateral, dislocating force. The patella is stabilized by the vastus medialis muscle Females are at a greater risk of dislocating the patella due to the width of their hips, which tends to cause genu valgum ("knocked-knee" appearance)

A 32-year-old mixed martial arts fighter could not continue his fight after receiving a side leg kick to the neck of his left fibula. The fighter reported paresthesia and numbness on the entire dorsum of his left foot. During his physical examination, the patient often stumbled with his left toes dragging on the floor during the swing phase of his gait. Asymmetry in his normal foot position was also noted by the physician as well as weakness in eversion of the foot at the ankle joint. What nerve was damaged? (A) Tibial nerve (B) Deep fibular nerve (C) Superficial fibular nerve (D) Common fibular nerve (E) Sciatic nerve

The answer is D: Common fibular nerve. The common fibular (peroneal) nerve, a terminal branch of the sciatic nerve, courses around the neck of the fibula. It is at this site that this nerve is particularly prone to injury via trauma, such as a kick to the side of the leg. Distal to this location, the common fibular nerve divides into the deep and superficial fibular nerves, which supply the motor innervation to the anterior and lateral compartments of the leg, respectively. Damage to the common fibular nerve would lead to all of the symptoms seen in this patient, such as weakness in eversion and dorsiflexion of the foot at the ankle joint, foot drop, and loss of sensation to the dorsum of the foot.

Human feet are everted so that their soles lie fully on the ground during ambulation. What muscle is developmentally unique to humans, inserts into the base of the fifth metatarsal, and assists in eversion (or pronation)? (A) Extensor digitorum longus (B) Extensor digitorum brevis (C) Fibularis longus (D) Fibularis tertius (E) Flexor digiti minimi brevis

The answer is D: Fibularis tertius. The fibularis (peroneus) tertius muscle arises off the distal portion of the extensor digitorum longus muscle and attaches to the dorsal surface of the base of the fifth metatarsal. This muscle is rarely found in other primates and is functionally linked to efficient terrestrial bipedalism. Though this muscle may be highly variable in human specimens, it is important in eversion of the foot during ambulation and may have a proprioceptive role to guard against excessive inversion of the foot at the ankle joint

A 25-year-old man suffers a gunshot wound to the calf that severs the posterior tibial artery at its origin. Which of the following vessels will not receive blood fl ow immediately following the injury? (A) Anterior tibial artery (B) Inferior medial genicular artery (C) Dorsalis pedis artery (D) Popliteal artery (E) Fibular (peroneal) artery

The answer is E: Fibular (peroneal) artery. The posterior tibial artery supplies the posterior compartments of the leg and the plantar aspect of the foot. A major branch is the fibular (peroneal) artery, which supplies the fibular side of the posterior compartment of the leg. The popliteal artery terminates by dividing into the anterior and posterior tibial arteries.

A 55-year-old woman recently had pelvic surgery during which cancerous lymph nodes were removed from the lateral wall of her pelvis. During a postoperative examination, she says she has been having painful muscle spasms in her thigh. Which of the following muscles is most likely involved? (A) Sartorius (B) Biceps femoris (C) Tensor muscle of fascia lata (D) Vastus medialis (E) Gracilis

The answer is E: Gracilis. The obturator nerve crosses the lateral wall of the pelvis on its way to the obturator canal and into the lower limb, and may be damaged in surgery or trauma to the lateral pelvic wall. Ultimately, it supplies the muscles in the medial compartment of the thigh, including the gracilis. Trauma to the efferent fibers within the obturator nerve may produce spasms or dysfunction in any of these muscles.

As part of a physical examination to evaluate lower limb function, a physician asks a patient to abduct her second through fifth toes. What specific nerve is the doctor testing? (A) Sural nerve (B) Deep fibular nerve (C) Superficial fibular nerve (D) Medial plantar nerve (E) Lateral plantar nerve

The answer is E: Lateral plantar nerve. The lateral plantar nerve innervates the dorsal interossei of the foot and the abductor digiti minimi muscles, which are responsible for abduction of the lateral four toes. The sural nerve is a cutaneous nerve, so it would not be responsible for abduction of the lateral four toes. The course of this nerve parallels the small saphenous vein, and damage to the sural nerve would lead to numbness and paresthesia in the dorsal aspect of the lateral fifth toe and lateral malleolus of the fibula. No motor deficits would be seen. The deep fibular (peroneal) nerve supplies motor innervation to the four muscles of the anterior compartment of the leg and two intrinsic muscles of the foot: extensor digitorum brevis and extensor hallucis brevis. The superficial fibular nerve supplies motor innervation to the two muscles (fibularis longus and brevis) of the lateral compartment of the leg and cutaneous (sensory) innervation to the distal third of the anterior surface of the leg and dorsum of the foot. These muscles are responsible for eversion. Though it would be responsible for abduction of the first toe, the medial plantar nerve is not responsible for abduction of the lateral four toes, which are being tested in this patient.

The deep peroneal nerve supplies:

The anterior muscles of the leg, including the tibialis anterior, extensor hallucis longus, extensor digitorum longus, and peroneus tertius muscles, which dorsiflex the foot

A 15-year-old girl, unaware that she is pregnant, borrows her friend's Accutane (retinoic acid; vitamin A) to combat an acne problem. She uses the Accutane for about 2 months, which corresponds to weeks 7 to 15 of the embryonic development of her fetus. Which of the following skeletal elements is most likely to be absent in the newborn infant? (A) Ilium (B) Femur (C) Patella (D) Tibia (E) Phalanges

The answer is E: Phalanges. Accutane (retinoic acid; vitamin A) is a useful therapeutic drug in relieving skin disorders such as acne. However, it also is a recognized teratogenic agent that should not be utilized during pregnancy. Both upper and lower limbs follow the same pattern of development, except that corresponding events occur slightly later (by ~2 days) in the lower limb. Overall, the major formative events take place from late week 4 to week 8. In this case, the Accutane was used during the later stages of limb development. The structures most likely affected are the most distal elements, the phalanges.

A veteran infantry soldier develops painful fl at feet after several years of service including hundreds of miles of marches. The pain is particularly acute on the medial aspect of his sole. Which of the following structures is most likely strained in this condition? (A) Calcaneal (Achilles) tendon (B) Plantar calcaneocuboid (long plantar) ligament (C) Extensor retinaculum (D) Tendon of the fibularis (peroneus) longus muscle (E) Plantar calcaneonavicular (spring) ligament

The answer is E: Plantar calcaneonavicular (spring) ligament. The condition of "flat-foot" ("fallen arch") is that in which the medial longitudinal arch of the foot significantly lowers or collapses altogether. This collapse may cause pain due to stretching of the plantar muscles and ligaments.

A 35-year-old male prisoner received a right gluteal intramuscular (IM) injection during a visit to the infirmary. Following the injection, the man experienced a painful, swollen right leg. Within a month, he complained that his right leg started to shrink. Examination revealed muscle wasting with fasciculations in the L4-S1 distribution and marked weakness in dorsiflexion, inversion, and eversion at the ankle joint. He also exhibited a typical high-steppage gait indicating right foot drop. What nerve was most likely damaged during the gluteal IM injection? (A) Superior gluteal nerve (B) Common fibular nerve (C) Tibial nerve (D) Inferior gluteal nerve (E) Sciatic nerve

The answer is E: Sciatic nerve. The sciatic nerve is formed by the anterior rami of L4-S3 and is located in the inferior medial quadrant of the buttock. This nerve supplies the muscles in the posterior compartment of the thigh and bifurcates into the tibial nerve and common fibular nerves in the popliteal fossa. The patient exhibits damage to the L4-S1 distribution of this nerve, which would severely limit inversion (L4, L5), eversion (L5, S1), and dorsiflexion (L4, L5) of the foot at the ankle joint. *The inability to dorsiflex the foot also has led to foot drop.*

A 32-year-old male farmer cuts the medial aspect of his midthigh when climbing over a barbwire fence. Though he bandages the wound, he reports to the ER 5 days later with an infected wound, high fever (102.7°), and lymphadenitis (swollen lymph nodes). Given the location of the injury, which groups of nodes would be the first to receive drainage from the infected wound? (A) Popliteal (B) External iliac (C) Deep inguinal (D) Horizontal group of superfi cial inguinal (E) Vertical group of superfi cial inguinal

The answer is E: Vertical group of superficial inguinal. The vertical group of superfi cial inguinal lymph nodes receives superficial lymph vessels from the territory drained by the great saphenous vein (including the medial aspect of the thigh) and lies along its termination near the saphenous hiatus

Which muscle can both dorsiflex and invert her foot?

The extensor hallucis longus can dorsiflex and invert the foot.

KNOW THIS:

The inferior gluteal nerve innervates the gluteus maximus, and the nerve to the obturator internus supplies the obturator internus and superior gemellus muscles. The obturator nerve innervates the adductor muscles of the thigh, and the femoral nerve supplies the flexors of the thigh.

Muscles that can evert the foot:

The peroneus longus, brevis, and tertius and extensor digitorum longus

KNOW THIS:

The popliteal vein drains blood into the femoral vein; thus, blood flow in the femoral vein is reduced. The great saphenous vein drains into the upper part of the femoral vein. Other veins (e.g. Posterior tibial & Anterior tibial & Lesser saphenous) empty into the popliteal vein.

The femoral nerve innervates:

The quadratus femoris, sartorius, and vastus muscles.

KNOW THIS:

The saphenous nerve, a sensory branch of the femoral nerve, supplies the skin and subcutaneous tissue on the anterior and medial aspects of the knee, leg, and foot. The superficial fibular nerve is a terminal branch of the common fibular nerve that supplies innervation to the lateral compartment of the leg. The obturator nerve supplies motor innervation to the muscles of the medial compartment of the thigh, including the medial half of the Pectineus muscle, Obturator externus, Gracilis, and the Adductor muscles: adductor brevis, adductor longus, and the medial (adductor) half of the adductor magnus. The tarsus, or proximal foot, consists of seven bones: Talus, Calcaneus, Navicular, Medial cuneiform, Intermediate cuneiform, Lateral cuneiform, and Cuboid. A mnemonic for the tarsal bones is "Tiger Cubs Need MILC," The talus (L: ankle bone) is located between the malleoli of the fibula and tibia and is the only tarsal bone without tendinous or muscular attachments.

KNOW THIS:

The sartorius can flex and rotate the thigh laterally, and flex and rotate the leg medially. The rectus femoris flexes the thigh and extends the leg. The semimembranosus extends the thigh and flexes and rotates the leg medially. The biceps femoris extends the thigh and flexes and rotates the leg laterally. The adductor longus adducts and flexes the thigh.

KNOW THIS:

The semitendinosus extends the thigh and flexes the leg. The short head of the biceps femoris flexes the leg. The adductor magnus adducts, flexes, and extends the thigh. The sartorius and gracilis can flex the thigh and leg.

KNOW THIS:

The tibialis anterior muscle can dorsiflex the foot

Posterior Femoral Cutaneous Nerve

■ Arises from the sacral plexus (S1-S3), passes through the greater sciatic foramen below the piriformis muscle, runs deep to the gluteus maximus muscle, and emerges from the inferior border of this muscle. ■ Descends in the posterior midline of the thigh deep to the fascia lata and pierces the fascia lata near the popliteal fossa. ■ Innervates the skin of the buttock, thigh, and calf.

Iliotibial Tract

■ Helps form the fibrous capsule of the knee joint and is important in maintaining posture and locomotion

Lateral Sural Cutaneous Nerve

■ Arises from the common peroneal nerve in the popliteal fossa and may have a communicating branch that joins the medial sural cutaneous nerve. ■ Innervates the skin on the posterolateral side of the leg.

Saphenous Nerve

■ Arises from the femoral nerve in the femoral triangle and descends with the femoral vessels through the femoral triangle and the adductor canal. ■ Becomes cutaneous between the sartorius and the gracilis and descends behind the condyles of the femur and tibia and medial aspect of the leg in company with the great saphenous vein. ■ Innervates the skin on the medial side of the leg and foot. ■ Is vulnerable to injury (proximal portion) during surgery to repair varicose veins.

Lateral Femoral Cutaneous Nerve

■ Arises from the lumbar plexus (L2-L3), emerges from the lateral border of the psoas major, crosses the iliacus, and passes under the inguinal ligament near the anterior-superior iliac spine. ■ Innervates the skin on the anterior and lateral aspects of the thigh as far as the knee.

Femoral Triangle

■ Is bounded by the inguinal ligament superiorly, the sartorius muscle laterally, and the adductor longus muscle medially. ■ Has the floor, which is formed by the iliopsoas, pectineus, and adductor longus muscles. Its roof is formed by the fascia lata and the cribriform fascia. ■ Contains the femoral nerve, artery, vein, and lymphatics (in the canal). A mnemonic NAVeL is used to remember the order of the structures, and the mnemonic NAVY is used to remember the structures from lateral to medial (nerve, artery, vein, yahoo!!!). The pulsation of the femoral artery may be felt just inferior to the midpoint of the inguinal ligament.

Sural Nerve

■ Is formed by the union of the medial sural and lateral sural nerves (or the communicating branch of the lateral sural nerve). ■ Innervates the skin on the back of the leg and the lateral side of the ankle, heel, and foot.

2. Calcaneus

■ Is the largest and strongest bone of the foot and lies below the talus. ■ Forms the heel of the foot, articulates with the talus superiorly and the cuboid anteriorly, and provides an attachment for the Achilles tendon.

Common Peroneal (Fibular) Nerve (L4-S2)

■ Is vulnerable to injury as it winds around the neck of the fibula, where it also can be palpated.

Femoral Artery

■ Is vulnerable to injury because of its relatively superficial position in the femoral triangle. ■ Includes several branches: -------------------------------- 1. Superficial Epigastric Artery 2. Superficial Circumflex Iliac Artery 3. Superficial External Pudendal Artery 4. Deep External Pudendal Artery 5. Profunda Femoris (Deep Femoral) Artery 6. Medial Femoral Circumflex Artery 7. Lateral Femoral Circumflex Artery 8. Descending Genicular Artery

1. Talus

■ Transmits the weight of the body from the tibia to the foot and is the only tarsal bone without muscle attachments. Has a body with a groove on its posterior surface for the flexor hallucis longus tendon.


Kaugnay na mga set ng pag-aaral

Intestinal Obstructions (Sherpath)

View Set

Information Security Fundamentals

View Set

Property & Casualty Insurance Fundamentals

View Set

Social Studies Daily Life in Athens vs Sparta

View Set

Ch 19: Documenting and Reporting

View Set

Chapter 6 Government Based on Study Guide

View Set

Chapter 10 - Acquisition and Disposition of Property, Plant, and Equipment

View Set

Chapter 2 from Intro to Business (ethnics)

View Set